anat 50q.doc

89
USMLE WORLD STEP 1 ANATOMY Question List Anatomy Q No: 1 Neurology Anatomy Q No: 43 Neurology Anatomy Q No: 2 Musculoskeletal Anatomy Q No: 44 Musculoskeletal Anatomy Q No: 3 Musculoskeletal Anatomy Q No: 45 Musculoskeletal Anatomy Q No: 4 Pulmonology Anatomy Q No: 46 Gastrointestinal system Anatomy Q No: 5 Neurology Anatomy Q No: 47 Reproductive system Anatomy Q No: 6 Neurology Anatomy Q No: 48 Head and neck Anatomy Q No: 7 Musculoskeletal Anatomy Q No: 49 Gastrointestinal system Anatomy Q No: 8 Neurology Anatomy Q No: 50 Musculoskeletal Anatomy Q No: 9 Head and neck Anatomy Q No: 51 Genitourinary Anatomy Q No: 10 Cardiology Anatomy Q No: 52 Neurology Anatomy Q No: 11 Genitourinary Anatomy Q No: 53 Neurology Anatomy Q No: 12 Neurology Anatomy Q No: 54 Blood vessels Anatomy Q No: 13 Neurology Anatomy Q No: 55 Musculoskeletal Anatomy Q No: 14 Neurology Anatomy Q No: 56 Neurology Anatomy Q No: 15 Head and neck Anatomy Q No: 57 Neurology Anatomy Q No: 16 Genitourinary Anatomy Q No: 58 Neurology Anatomy Q No: 17 Gastrointestinal system Anatomy Q No: 59 Neurology Anatomy Q No: 18 Pulmonology Anatomy Q No: 60 Neurology Anatomy Q No: 19 Musculoskeletal Anatomy Q No: 61 Cardiology Anatomy Q No: 20 Neurology Anatomy Q No: 62 Pulmonology Anatomy Q No: 21 Cardiology Anatomy Q No: 63 Hepatobiliary system Anatomy Q No: 22 Genitourinary Anatomy Q No: 64 Head and neck Anatomy Q No: 23 Genitourinary Anatomy Q No: 65 Musculoskeletal Anatomy Q No: 24 Pulmonology Anatomy Q No: 66 Genitourinary Anatomy Q No: 25 Head and neck Anatomy Q No: 67 Gastrointestinal system 1

Upload: dodakadeja

Post on 01-Jan-2016

194 views

Category:

Documents


6 download

DESCRIPTION

frf

TRANSCRIPT

Page 1: Anat 50q.doc

USMLE WORLD STEP 1 ANATOMY

Question List

Anatomy Q No: 1 Neurology Anatomy Q No: 43 Neurology

Anatomy Q No: 2 Musculoskeletal Anatomy Q No: 44 Musculoskeletal

Anatomy Q No: 3 Musculoskeletal Anatomy Q No: 45 Musculoskeletal

Anatomy Q No: 4 Pulmonology Anatomy Q No: 46 Gastrointestinal system

Anatomy Q No: 5 Neurology Anatomy Q No: 47 Reproductive system

Anatomy Q No: 6 Neurology Anatomy Q No: 48 Head and neck

Anatomy Q No: 7 Musculoskeletal Anatomy Q No: 49 Gastrointestinal system

Anatomy Q No: 8 Neurology Anatomy Q No: 50 Musculoskeletal

Anatomy Q No: 9 Head and neck Anatomy Q No: 51 Genitourinary

Anatomy Q No: 10 Cardiology Anatomy Q No: 52 Neurology

Anatomy Q No: 11 Genitourinary Anatomy Q No: 53 Neurology

Anatomy Q No: 12 Neurology Anatomy Q No: 54 Blood vessels

Anatomy Q No: 13 Neurology Anatomy Q No: 55 Musculoskeletal

Anatomy Q No: 14 Neurology Anatomy Q No: 56 Neurology

Anatomy Q No: 15 Head and neck Anatomy Q No: 57 Neurology

Anatomy Q No: 16 Genitourinary Anatomy Q No: 58 Neurology

Anatomy Q No: 17 Gastrointestinal system Anatomy Q No: 59 Neurology

Anatomy Q No: 18 Pulmonology Anatomy Q No: 60 Neurology

Anatomy Q No: 19 Musculoskeletal Anatomy Q No: 61 Cardiology

Anatomy Q No: 20 Neurology Anatomy Q No: 62 Pulmonology

Anatomy Q No: 21 Cardiology Anatomy Q No: 63 Hepatobiliary system

Anatomy Q No: 22 Genitourinary Anatomy Q No: 64 Head and neck

Anatomy Q No: 23 Genitourinary Anatomy Q No: 65 Musculoskeletal

Anatomy Q No: 24 Pulmonology Anatomy Q No: 66 Genitourinary

Anatomy Q No: 25 Head and neck Anatomy Q No: 67 Gastrointestinal system

Anatomy Q No: 26 Genitourinary Anatomy Q No: 68 Gastrointestinal system

Anatomy Q No: 27 Musculoskeletal Anatomy Q No: 69 Neurology

Anatomy Q No: 28 Musculoskeletal Anatomy Q No: 70 Genitourinary

Anatomy Q No: 29 Neurology Anatomy Q No: 71 Genitourinary

Anatomy Q No: 30 Neurology Anatomy Q No: 72 Gastrointestinal system

Anatomy Q No: 31 Musculoskeletal Anatomy Q No: 73 Head and neck

Anatomy Q No: 32 Renal Anatomy Q No: 74 Musculoskeletal

Anatomy Q No: 33 Neurology Anatomy Q No: 75 Musculoskeletal

Anatomy Q No: 34 Head and neck Anatomy Q No: 76 Gastrointestinal system

Anatomy Q No: 35 Musculoskeletal Anatomy Q No: 77 Neurology

Anatomy Q No: 36 Musculoskeletal Anatomy Q No: 78 Neurology

Anatomy Q No: 37 Neurology Anatomy Q No: 79 Renal

Anatomy Q No: 38 Genitourinary Anatomy Q No: 80 Pulmonology

Anatomy Q No: 39 Pulmonology Anatomy Q No: 81 Musculoskeletal

Anatomy Q No: 40 Musculoskeletal Anatomy Q No: 82 Genitourinary

Anatomy Q No: 41 Head and neck Anatomy Q No: 83 Gastrointestinal system

Anatomy Q No: 42 Neurology Anatomy Q No: 84 Musculoskeletal

Anatomy Q No: 85 Genitourinary Anatomy Q No: 89 Musculoskeletal

Anatomy Q No: 86 Gastrointestinal system Anatomy Q No: 90 Musculoskeletal

1

Page 2: Anat 50q.doc

USMLE WORLD STEP 1 ANATOMY

Anatomy Q No: 87 Neurology Anatomy Q No: 91 Musculoskeletal

Anatomy Q No: 88 Musculoskeletal Anatomy Q No: 92 Gastrointestinal system

2

Page 3: Anat 50q.doc

USMLE WORLD STEP 1 ANATOMY

A. Between the medulla and the cerebellar hemispheres B. Above the diaphragma sellae C. Over the lateral hemispheric fissure D. Between the cerebellum and the lateral pans E. In the parasagittal aspect of the brain convexity F. Between the cerebellar peduncles

Explanation:Intracranial schwannomas are most commonly located at the cerebropontine angle, which is between the cerebellum and lateral pons. Schwannomas of this location arise from CN VIII and are called acoustic neuromas. CN VII (facial nerve) and CN V (trigeminal nerve) are in proximity to CN VIII and may also be affected by acoustic neuromas via compression. The patient in this vignette displays symptoms of CN VIII involvement as well as CN V and CN VII. 1. The cochlear part of CN VIII mediates hearing. Its compression leads to sensorineural hearing loss and tinnitus (ear ringing). 2. The vestibular part of CN VIII maintains balance. Its compression causes vertigo, disequilibrium, and nystagmus. 3. CN VII involvement leads to paralysis of the muscles of facial expression, loss of taste in the anterior 2/3 of the tongue, and hyperacusis (paralysis of the stapedius). 4. Compression of CN V causes loss of sensation around the mouth and nose, loss of corneal reflex, and paralysis of the muscles of mastication. Bilateral acoustic neuromas are associated with neurofibromatosis type 2. (Choice B) Germ cell tumors arise in the pineal and suprasellar regions. They can cause aqueductal stenosis (with obstructive hydrocephalus) and Parinaud syndrome (paralysis of vertical gaze). (Choices C and E) Meningiomas are commonly found over the lateral hemispheric fissure and in the parasagittal aspect of the brain convexity. On light microscopy, meningiomas have a whorled pattern of cell growth—also, psammoma bodies are present. Clinically, meningiomas may manifest with headache focal deficit, or seizure.

Educational Objective: Sensorineural hearing loss, tinnitus, paralysis of facial muscles, and loss of corneal reflex signify the involvement of CN V, VII, and VIII. Simultaneous compression of these nerves is caused by tumor of the cerebellopontine angle, which are most commonly acoustic neuromas.

3

Q NO 1: A 34-year-old female presents to your office with moderate hearing loss that she noticed several months ago. She also complains of a noise in her right ear that is very disturbing. Physical examination shows right-sided hearing loss, an asymmetric smile, and decreased corneal reflex in her right eye. This patient’s condition suggests an intracranial mass located in which of the following locations?

Page 4: Anat 50q.doc

USMLE WORLD STEP 1 ANATOMY

A.

Abduction of the thigh B. Adduction of the thigh C. Flexion of the thigh D. Extension of the thigh E. Extension of the leg

Explanation:The obturator nerve is the only nerve that exits the pelvis via the obturator foramen. This nerve innervates the adductor compartment of the thigh after dividing into anterior and posterior branches subsequent to leaving the pelvis. The anterior branch innervates the gracilis, pectineus, and the adductors longus and brevis. The posterior branch innervates the obturator externus and the adductor magnus. Obturator nerve injury would cause weakness and spasm of the adductor compartment muscles. (Choice A) Abduction of the thigh is accomplished by the tensor fascia lata and the sartorius muscles. These muscles are supplied by the superior gluteal and femoral nerves, respectively, and lie in the anterior compartment of the thigh. (Choice C) Flexion of the thigh is accomplished by the psoas, iliacus, tensor fascia lata, and the sartorius muscles. The psoas is directly innervated by the lumbar plexus, and the iliacus is innervated by the femoral nerve. (Choice D) Extension of the thigh is accomplished primarily by the gluteus maximus muscle, which is supplied by the inferior gluteal nerve. This nerve exits the pelvis through the greater sciatic foramen below the piriformis. (Choice E) Extension of the leg is primarily accomplished by the quadriceps femoris muscle group. These muscles are innervated by the femoral nerve, which gains access to the thigh deep to the center of the inguinal ligament.

Educational Objective: The obturator nerve is the only major nerve that exits the pelvis through the obturator foramen. This nerve supplies the muscles of the medial (adductor) compartment of the thigh and can be damaged during pelvic surgery, especially in procedures such as lymph node dissection.

4

Q NO 2: A 56-year-old male with advanced bladder carcinoma suffers from compression of a nerve that passes through the obturator foramen. Which of the following functions would be most affected in this patient?

Page 5: Anat 50q.doc

USMLE WORLD STEP 1 ANATOMY

A. AB. BC. CD. DE. E

Explanation:The patient described in the question stem is experiencing paroxysmal supraventricular tachycardia. Treatment can be with vagal stimulation, such as carotid sinus massage or the Valsalva maneuver. Maneuvers that increase vagal tone increase the refractory period in the AV node and help prevent a reentrant circuit from conducting. If Valsalva measures fail, intravenous administration of adenosine is recommended. The Valsalva maneuver is executed by forcibly exhaling against a closed glottis. This is done by taking a full inhalation, closing the glottis (i.e. holding one’s breath), and subsequently bearing down—without exhaling—as one would during a bowel movement. The rectus muscles are recruited in this process, and they (Choice A) play the largest role in the development of the resultant elevated intraabdominal and intrathoracic pressure during this maneuver. (Choice B) This structure is the iliacus muscle a flexor oft he hip that lies over the iliac fossa. (Choice C) This structure is the gluteus minimus muscle. This muscle is innervated by the superior gluteal nerve and has the subtle, but important function of preventing the contralateral (non-weight-bearing) side of the pelvis from dipping when that leg is elevated off of the ground, as when one walks. (Choice D) This structure is the gluteus medius muscle. This muscle has the same innervation and function as the gluteus minimus. (Choice E) This structure is the gluteus maximus muscle. This muscle is innervated by the inferior gluteal nerve and is the major extensor of the thigh at the hip.

Educational Objective: The Valsalva maneuver increases vagal tone and can be used to abolish paroxysmal supraventricular tachycardia. The rectus abdominis is the most important muscle in achieving the increased intraabdominal and intrathoracic pressure of the Valsalva maneuver.

5

Q NO 3: A 32-year-old male presents to the ER with sudden-onset heart palpitations. His blood pressure is 100/70 mmHg, and his heart rate is 160/mm with regular rhythm. The physician instructs the patient to do the “Valsalva maneuver” to relieve these symptoms. Which of the following structures indicated on the pelvic CT image below is most important in performing the Valsalva maneuver?

Page 6: Anat 50q.doc

USMLE WORLD STEP 1 ANATOMY

A. Right lower lung lobe B. Liver C. Intercostal artery D. Intercostal nerve E. Hepatic veins

Explanation:To avoid complications while performing a thoracentesis, it is necessary to remember the location of the lungs, pleura, and other organs of the chest and upper abdomen. The portion of pleura that covers the surface of the lung is called the visceral pleura. The parietal pleura lines the inner surface of the chest wall and the diaphragm, and is innervated by somatic sensory nerves. The diaphragm is bound to the inferior margin of the thorax, and the portion of parietal pleura that covers the diaphragm extends to the following levels:

The lower border of the lung is usually located two intercostal spaces above the respective pleural border. Thoracentesis, therefore, should be performed between 5th and 7th ribs along the midclavicular line, the 7th and 9th ribs along the midaxillary line, and 9th and 11th ribs along the paravertebral line. If the needle is inserted higher there is a risk of lung injury. Insertion of the needle below the 9th rib at the middle axillary line on the right may cause liver injury (Choice B). (Choice A) The lower border of the right lung is located two intercostal spaces above the pleural border. The lung is not likely to be injured by insertion of a needle into the 10th intercostal space at the midaxillary line as described. (Choices C and D) The intercostal vein, artery, and nerve lie in the subcostal groove on the lower border of the rib. Thoracentesis should be performed along the upper border of the rib to prevent injury to these structures. (Choice E) Hepatic veins are not likely to be injured during thoracocentesis.

Educational Objective: Thoracocentesis should be performed above the 7th rib in midclavicular line, the 9th rib along midaxillary line and the 11th rib along posterior scapular line. Insertion of a needle lower than these points increases the risk of penetrating abdominal structures, and insertion of the needle on the inferior margin of the rib risks striking the subcostal neurovascular bundle.

6

Q NO 4: A medical resident performs a diagnostic procedure on 32-year-old male. He inserts a needle along the upper border of the 10th rib at the right midaxillary line. Which of the following is most likely to be punctured as a result of the intervention?

Page 7: Anat 50q.doc

USMLE WORLD STEP 1 ANATOMY

A.

Infection B. Tumor C. Muscle hypertonicityD. Lower extremity hyporeflexia E. Subarachnoid hemorrhage

Explanation:Hydrocephalus is any increase in cerebrospinal fluid (CSF) volume. In infancy and early childhood, the cause is usually impaired CSF outflow due to congenital abnormalities like Type II Arnold-Chiari malformations, hereditary aqueductal stenosis, and prenatal infections (e.g. toxoplasmosis). Hydrocephalus in early infancy typically presents with macrocephaly (defined as head circumference greater than two standard deviations above the mean for gender and age), irritability, poor feeding, muscle hypertonicity and hyperreflexia. Hypertonicity and hyperreflexia result from upper motor neuron damage caused by stretching of the periventricular pyramidal tracts. (Choices A, B and E) Infection, tumor, and subarachnoid hemorrhage are potential causes of acquired hydrocephalus but do not result from hydrocephalus. (Choice D) Lower extremity hyporeflexia would result from damage to the lower motor neurons of the lower extremities, as can occur in poliomyelitis or Werdnig-Hoffman disease.

Educational Objective: Hydrocephalus in infants presents with irritability, poor feeding increased head circumference and enlarged ventricles on CT. Long-term sequelae of hydrocephalus include lower extremity spasticity due to stretching of the periventricular pyramidal tracts, visual disturbances, and learning disabilities.

7

Q NO 5: A 6-month-old female has a head circumference that is in the 97th percentile for her age. CT scan of the head reveals significant dilation of the lateral ventricles. Which of the following is the most likely long-term complication of this patient’s condition?

Page 8: Anat 50q.doc

USMLE WORLD STEP 1 ANATOMY

A. Hypoglossal B. Facial C. Accessory D. Vagal E. Glossopharyngeal

Explanation:The middle ear cavity contains three auditory ossicles (malleus, incus and stapes) and two skeletal muscles (tensor tympani and stapedius) that participate in the transmission of sound from the tympanic membrane to the inner ear. The stapedius muscle arises from the wall of the tympanic cavity and inserts on the neck of the stapes. The body’s smallest striated muscle, its function is to stabilize the stapes, the body’s smallest bone. The stapedius muscle is innervated by the stapedius nerve, a branch of facial nerve (CN VII). Paralysis of the stapedius muscle allows the stapes to oscillate more widely, producing hyperacusis (increased sensitivity to sound). Ipsilateral hyperacusis is a common finding in Bell’s palsy (peripheral facial nerve paralysis). Other findings in facial nerve paralysis include inability to close the eye or to smile on the affected side, as well as ipsilateral increased salivation and loss of taste on the anterior 2/3 of the tongue. Injury to the other cranial nerves listed in the answer choices does not lead to hyperacusis. Note also that the tensor tympani muscle arises from the cartilaginous portion of the auditory tube and the adjoining part of the sphenoid bone and inserts into the malleus. The tensor tympani draws the tympanic membrane medially, increasing its tension. This muscle is innervated by the mandibular branch of the trigeminal nerve (CN V3).

Educational Objective: The stapedius muscle is innervated by the stapedius nerve, a branch of facial nerve (CN VII). Paralysis of the stapedius muscle allows wider oscillation of the stapes, and leads to increased sensitivity to sound (hyperacusis).

8

Q NO 6: A 34-year-old male is unable tolerate everyday sounds. Injury of which of the following cranial nerves might be responsible?

Page 9: Anat 50q.doc

USMLE WORLD STEP 1 ANATOMY

A. Common peroneal B. Femoral C. Tibial D. Radial E. Pudendal E. Lateral sural cutaneous

Explanation:The common peroneal nerve is the lateral branch of the sciatic nerve and originates at the superior aspect of the popliteal fossa. It then traces the lateral aspect of the fossa, crosses the head of the fibula, and comes into direct contact with the lateral neck of the fibula as it enters the leg. At this point it divides into the deep peroneal nerve and superficial peroneal nerve. The deep peroneal innervates the extensor and the great dorsiflexors. The superficial peroneal branch supplies the peroneal muscles and the skin of most of the toes. The common peroneal nerve is particularly susceptible to damage as it traces the lateral neck of the fibula. Damage typical occurs via compression or leg fracture. The result is inability to dorsiflex the foot. (Choice B) The femoral nerve (L2-L4) is the largest nerve in the lumbar plexus and innervates the muscles of the anterior and medial thigh. Damage to the femoral nerve will result in loss of knee jerk. (Choice C) The tibial nerve is the larger of the two terminal branches of the sciatic nerve and it provides motor innervation for the popliteus and the flexors of the foot. Damage of this nerve would yield difficulty with plantar flexion. (Choice D) The radial nerve innervates the upper extremity and damage results in wrist drop. (Choice E) The pudendal nerve provides the majority of the motor and sensory input to the pelvic floor. (Choice F)The lateral sural cutaneous is a cutaneous branch of the tibial nerve and supplies sensation to the calf.

Educational Objective: Common peroneal nerve injury as it traces the lateral aspect of the femoral neck is common. Bony facture and compression are the most common causes. Clinically this manifests as foot drop.

9

Q NO 7: A 54 year-old alcoholic who has been sober for 6 months “falls of the wagon” and imbibes a large amount of isopropyl alcohol. After an unknown period of time she is found by her neighbor and brought into the emergency room. The patient eventually arouses but is troubled by a persistent right foot drop. Prolonged compression of what nerve most likely accounts for her symptoms?

Page 10: Anat 50q.doc

USMLE WORLD STEP 1 ANATOMY

A.Tibial B. Common peroneal C. Superficial peroneal D. Sural E. Obturator

Explanation:The common peroneal nerve is the most commonly injured nerve in the leg. It begins in the proximal popliteal fossa, where the sciatic nerve divides into the tibial nerve and common peroneal nerve. From there, the common peroneal nerve courses laterally into the anterior compartment of the leg, passing superficially around the head and neck of the fibula. Blunt trauma to the lateral leg and fractures of the neck of the fibula can injure this nerve. Patients with common peroneal nerve damage present with an equinovarus (plantarflexed and inverted) posture of the affected foot due to paralysis of the peroneus longus and peroneus brevis muscles (mediate foot eversion), paralysis of the tibialis anterior muscle (mediates dorsiflexion), and paralysis of the extrinsic extensors of the toes. Injury to this nerve also causes loss of sensation to the anterolateral leg. The classic finding on gait exam in patients with common peroneal nerve injury is ‘toot drop,” where the affected leg is lifted high off of the ground while walking due

10

Q NO 8: A patient presents to your office with right leg numbness. When he walks, you notice that he lifts his right foot higher than he does his left, and that his right foot slaps to the ground with each step. On neurologic exam, you also ascertain that he is unable to evert the right foot. Which of the following nerves has most likely been injured?

Page 11: Anat 50q.doc

USMLE WORLD STEP 1 ANATOMY

to an inability to dorsiflex the foot. The affected foot will also classically slap to the ground with each step. (Choice A) Tibial nerve injury causes dorsiflexion and eversion of the foot with sensory loss on the sole of the foot. (Choice C) The superficial peroneal nerve innervates the peroneus longus and peroneus brevis muscles in the lateral compartment of the leg, which together evert the foot. It also provides sensory innervation to the skin of the distal anterior leg and dorsal foot. (Choice E) Obturator nerve injury classically causes spasms of the adductor muscles of the thigh.

Educational Objective: Common peroneal nerve injury is common and typically results from trauma to the leg near the head of the fibula. Signs include foot drop and a characteristic high-stepping gait.

11

Page 12: Anat 50q.doc

USMLE WORLD STEP 1 ANATOMY

A. Tympanic membrane B. Middle ear ossicles C. Round window D. Organ of Coil E. Cupula F. Auditory nerve

Explanation:Transduction of mechanical auditory forces into nerve impulses occurs in the organ of Corti by the following steps: 1) Sound reaches the middle ear by vibrating the tympanic membrane. 2) This vibration is transferred to the oval window by the ossicles. 3) Vibration of the oval window causes vibration of the basilar membrane which in turn causes bending of the hair cell cilia against the tectorial membrane. 4) Hair cell bending causes oscillating hyperpolarization and depolarization of the auditory nerve, thus creating nerve impulses from sound. Noise-induced hearing loss results from trauma to the stereociliated hair cells of the organ of Corti. Prolonged exposure to noises greater than 85 dB can cause distortion or fracture of the stereocilia due to shearing forces against the tectorial membrane. High-frequency hearing is lost first, regardless of the frequency of the sound causing the damage. Initial damage causes loss of hearing at frequencies of approximately 4000 Hz (speech frequencies = 500-3000 Hz). The acoustic reflex dampens the effects of prolonged loud noise by causing the stapedius and tensor tympani muscles to contract, lessening the responsiveness of the ossicles to sound. (Choice A) Perforation or rupture of the tympanic membrane can occur with infection, trauma, pressure changes when the eustachian tube is blocked, or sudden very loud noises. Rupture of the tympanic membrane causes conductive hearing loss. (Choice B) Defects of the middle ear ossicles such as otosclerosis can cause conductive hearing loss. (Choice C) The round window lies between the middle and inner ear and moves outward when the stapes causes the oval window to move inward and vice versa. (Choice E)The cupula lies at the apex of the cochlea and houses the cells that sense rotation. (Choice F) The auditory nerve transmits sound impulses to the brainstem via the 8th cranial nerve. Defects in this nerve can result from trauma mass effect, demyelination, or local inflammatory processes, and can cause nerve-based hearing

Educational Objective: Prolonged exposure to loud noises causes hearing loss due to damage of the stereociliated hair cells of the organ of Corti.

12

Q NO 9: A 34-year-old male presents to your office complaining of decreased hearing in both ears. He is a rock musician and has been spending a lot of time in his studio recently. Where is the most likely location of ear injury in this patient?

Page 13: Anat 50q.doc

USMLE WORLD STEP 1 ANATOMY

A. Tricuspid valve B. Mitral valve C. Pulmonic valve D. Aortic valve E. Coronary sinus

Explanation:If the aim of cardiac catheterization is to measure left ventricular and aortic pressures, the cardiologist must pass the catheter into the left ventricle and the aorta. The fossa ovalis is an oval-shaped depression on the septal wall of the right atrium. It lies in the location of the obliterated fetal foramen ovale. The catheter entered via the femoral vein will follow the venous blood flow to the inferior vena cava that drains into the right atrium. By puncturing the fossa ovalis, the cardiologist will pass the catheter from the right atrium directly into the left atrium. To gain access to the left ventricle from the left atrium, the catheter will pass through the mitral valve (Choice B). See the diagram below for the catheter route:

(Choice A) The tricuspid valve separates the right atrium from the right ventricle. (Choice C) The pulmonic valve is located between right ventricle and the pulmonary artery.

13

Q NO 10: A 67-year-old male suffers from progressive exertional dyspnea and fatigue. Cardiac catheterization is planned to measure the left ventricular systolic and diastolic pressures as well as aortic pressures. The cardiologist decides to enter the right femoral vein and then access the left heart by puncturing the fossa ovalis. Which of the following cardiac structures will the catheter pass first during the procedure?

Page 14: Anat 50q.doc

USMLE WORLD STEP 1 ANATOMY

(Choice D) Aortic valve divides left ventricle and aorta. It is the second valve that will be passed by the catheter on the way from right atrium to aorta. (Choice E)The coronary sinus collects blood from the coronary veins. It is located on the posterior surface of the heart and drains directly into the left atrium.

Educational Objective: The fossa ovalis is located in the right atrial wall: it is the remnant of the fetal foramen ovale, a structure that allows right to left shunting of blood in the fetal circulation to bypass the fetal lungs. Puncture of the fossa ovalis is used as a means of gaining access to the left atrium from the right atrium. The route of the catheter mentioned in this question stem is as follows: right atrium— fossa ovale—left atrium—a mitral valve— left ventricle—a aortic valve—a aorta.

14

Page 15: Anat 50q.doc

USMLE WORLD STEP 1 ANATOMY

A. Inferior epigastric vessels B. Cooper’s ligament C. Femoral vein D. Transversalis fascia E. Rectus muscle sheath F. Spermatic cord

Explanation:There are three types of groin hernias: direct inguinal, indirect inguinal and femoral hernia. Direct and indirect inguinal hernias occur above the inguinal ligament, while femoral hernias are located below it. Femoral hernias protrude through the femoral canal. They are lateral to the pubic tubercle and lacunar ligament and medial to the femoral vein. Femoral hernias are more common in females, and tend to occur on the right side. Femoral hernias may present with upper thigh and groin pain. As the femoral canal is small, femoral hernias are prone to incarceration. Incarcerated hernias cannot be reduced; if a segment of bowel is present in the hernia sac, incarceration can cause bowel obstruction. Impaired blood flow to contents trapped in the hernia sac occurs a few hours after incarceration causing strangulation. Ischemia and necrosis of the contents of the hernia sac occur. (Choice A) Inferior epigastric vessels are located laterally to direct inguinal hernias and medially to indirect inguinal hernias. They constitute the lateral border of Hesselbach triangle. (Choice B) The pectineal (Cooper’s) ligament is a thickened part of the pectineal fascia. It overlies the pectineal ridge of the pubic bone and is located posteriorly to the femoral canal. (Choice D) Transversalis fascia is found between the inner surface of transversalis muscle and the extraperitoneal fat. It forms the posterior wall of the inguinal canal. A

15

Q NO 11: A 45-year-old Caucasian male presents to your office complaining of intermittent pain in his right thigh. Physical examination reveals a bulge below the inguinal ligament and just lateral to the pubic tubercle. On straining, the bulge increases in size. The bulge laterally contacts which of the following structures?

Page 16: Anat 50q.doc

USMLE WORLD STEP 1 ANATOMY

deep inguinal ring is an opening in the transversalis fascia, which is the site of protrusion of indirect inguinal hernias. (Choice E) The rectus muscle sheath forms the medial border of Hesselbach triangle. Direct inguinal hernias are located laterally to the rectus abdominis muscle sheath. (Choice F) The spermatic cord lies in the inguinal canal. It contains the ductus deferens, cremasteric, testicular arteries, artery of ductus deferens, pampiniform venous plexus, genitourinary nerve, and sympathetic and parasympathetic nerves of the spermatic plexus.

Educational Objective: Femoral hernias are inferior to the inguinal ligament lateral to pubic tubercle, and medial to the femoral vein. They manifest with a bulge on the upper thigh that enlarges with straining (Valsalva maneuver). Incarceration and strangulation are common complications of femoral hernia.

16

Page 17: Anat 50q.doc

USMLE WORLD STEP 1 ANATOMY

A. Tinnitus and unilateral deafness B. Headaches and anosmia C. Progressive leg weakness and spasticityD. Unilateral ptosis and gaze palsy E. Bitemporal visual field defects

Explanation:The image above shows a tumor adjacent to the falx cerebri and compressing the medial surface of the hemisphere. This is a typical meningioma location. Compression of the parasagittal cortex causes contralateral spastic paralysis of the leg. Focal seizures may also occur. Meningiomas are the second-most common brain tumors in adults (the most common is glioblastoma multiforme). These benign and slow-growing tumors arise from arachnoid villi. On light microscopy, meningiomas are composed of concentrically-arranged meningothelial cells. Psammoma bodies (concentric laminar calcifications) are characteristic. Meningiomas are commonly located in the parasagittal region: they can also be found adjacent to the lateral convexity of the hemisphere, in the region of the sphenoid wing and olfactory groove. (Choice A) Tinnitus and unilateral hearing loss are the common presenting features of a cerebellopontine angle tumor, such as an acoustic neuroma. (Choice B) Meningiomas located in the olfactory groove present with headache and anosmia. (Choice D) Unilateral ptosis and gaze palsy (“down and out” gaze) are features of CN Ill paralysis. CN Ill can suffer damage from compression (by berry aneurysm or uncal herniation) or via nerve ischemia (diabetes mellitus). (Choice E) Bitemporal hemianopia occurs due to compression of the central part of the optic chiasm. Pituitary adenoma and craniopharyngioma are common causes.

Educational Objective: Meningiomas are located adjacent to the cerebral surface. Parasagittal meningiomas cause contralateral spastic paresis of the leg due to compression of the leg-foot motor area.

17

Q NO 12: A 43-year-old female undergoes brain MRI for a long history of complex neurological complaints. The findings from the imaging study are shown below. You suspect that this patient initially suffered from which of the following?

Page 18: Anat 50q.doc

USMLE WORLD STEP 1 ANATOMY

A. Desmosomes B. Hemidesmosomes C. Intermediate junctions D. Fenestrae E. Gap junctions F. Tight junctions

Explanation:The experimental animal is unresponsive to an intravenous dopamine infusion due to the fact that intravenous dopamine is unable to cross the blood-brain barrier. Capillaries of the blood-brain barrier are not fenestrated, so paracellular passage of fluid and dissolved material does not occur in the CNS. The primary mediators of the blood-brain barrier are tight junctions between the endothelial cells of CNS capillaries. Tight junctions, also known as zonula occludens, are formed via the interaction of specialized transmembrane proteins with one another, such as occludens and claudens, on capillary endothelial cells. Due to tight junctions the only way that material can pass from the bloodstream to the brain is by transcellular movement, which is limited by the size and lipophilicity of the molecule and by carrier-mediated transport.

18

Q NO 13: Muscle rigidity is observed in an experimental animal that has chemically-destroyed dopaminergic neurons of the substantia nigra. The animal’s rigidity fails to improve with continuous dopamine infusion. Which of the following cell communications account for the lack of responsiveness to dopamine?

Page 19: Anat 50q.doc

USMLE WORLD STEP 1 ANATOMY

(Choice A) Desmosomes are referred to as “spot welds” and function as anchors holding adjoining cells together. They are most notable in the skin where they can be observed microscopically in the stratum spinosum. (Choice B) Hemidesmosomes are similar to desmosomes, but they serve to bind the basal layer of epithelial cells to the basement membrane. Diseases of the hemidesmosome in the skin include bullous pemphigoid and junctional epidermolysis bullosa. (Choice C) Intermediate junctions occur on surface epithelial cells below tight junctions and assist in cell-cell adhesion. (Choice D) Fenestrae are small pores in endothelial cells that allow free exchange of intravascular fluid with extravascular fluid in the tissue surrounding the capillary bed. (Choice E) Gap junctions are channels between cells that allow the free exchange of solute and fluid. They are typically formed by connexin proteins.

Educational Objective: The tight junctions between endothelial cells in the capillary beds of the CNS form the blood-brain barrier. Solutes and fluids cannot move freely across the capillary membrane in the CNS: material can only move transcellularly by diffusing across the epithelial plasma membranes or by carrier-mediated transport.

19

Page 20: Anat 50q.doc

USMLE WORLD STEP 1 ANATOMY

A. AB. BC. CD. DE. E

Explanation:The patient described in the question stem is most likely suffering from Huntington’s disease. This disease is an autosomal dominant condition that results from an excessive number of CAG trinucleotide repeats as compared to the unaffected population. Furthermore, families with this disease experience a process called amplification which is a progressive expansion of the trinucleotide repeats in successive generations leading to earlier expression of the disease in affected children of Huntington’s patients. Imaging studies typically show atrophy of the caudate, and metabolic imaging studies such as PET scanning show decreased metabolic activity in the striatum, which is composed of the caudate and putamen. The caudate nucleus (Choice A) is a curved or “C-shaped” structure in its complete form. The head of the caudate can be identified in the inferolateral walls of the anterior horns of the lateral ventricles. The head of the caudate nucleus is separated from the globus pallidus and putamen by the internal capsule. (Choice B) This structure is the internal capsule. The posterior limb of the internal capsule separates the globus pallidus and putamen from the thalamus and carries corticospinal motor and somatic sensory fibers as well as visual and auditory fibers. The function of this limb is the most important to know for the USMLE. The genu, or “knee”, of the internal capsule lies between the anterior and posterior branches and carries corticobulbar fibers. The anterior limb of the internal capsule separates the caudate nucleus from the globus pallidus and putamen and carries a portion of the thalamocortical fibers. (Choice C) This structure is the insular cortex (insula). The insula plays a role in the limbic system (emotion) as well as in the coordination of some autonomic functions, particularly of the cardiac system. (Choice D) This structure is the putamen. (Choice E) This structure is the globus pallidus.

Educational Objective: The head of the caudate lies in the inferolateral wall of the anterior horn of the lateral ventricle. It is separated from the globus pallidus and putamen by the internal capsule. The caudate is atrophied in Huntington’s disease.

20

Q NO 14: A 23-year-old Caucasian male with involuntary movements undergoes an experimental radioisotope study to localize metabolic abnormalities in his brain. The study shows abnormal metabolic activity in the caudate nucleus. Which of the following on the gross brain section slide below corresponds to the location of the abnormalities in this patient?

Page 21: Anat 50q.doc

USMLE WORLD STEP 1 ANATOMY

A.

Inferior orbital fissure B. Optic canal C. Superior orbital fissure D. Foramen rotundum E. Foramen lacerum F. Foramen ovale

Explanation:This patient’s diplopia is explained by his inability to adduct the left eye. Eye adduction depends on the oculomotor nerve (CN Ill) and the medial rectus muscle. The oculomotor nerve originates in the oculomotor nucleus of the midbrain, emerges from the anterior midbrain, and enters the orbit through the superior orbital fissure. The patient also has loss of the corneal reflex on the left side. The sensory limb of the corneal reflex is mediated by the nasociliary branch of the first division of the trigeminal nerve (CN V1). The motor component of the corneal reflex is carried primarily by the temporal branch of the facial nerve. Like the oculomotor nerve, the nasociliary nerve enters the orbit through the superior orbital fissure. An injury here would cause the deficits described. The trochlear nerve (CN IV), abducens nerve (CN VI), and superior ophthalmic vein also enter the orbit via the superior orbital fissure. (Choice A) The maxillary division of the trigeminal nerve, the infraorbital vessels, and branches from the sphenopalatine ganglion pass through the inferior orbital fissure but do not enter the orbit. (Choice B) The optic canal is medial to the superior orbital fissure and transmits only the optic nerve (CN II) and ophthalmic artery. (Choice D) The foramen rotundum transmits the maxillary division of the trigeminal nerve (CN V2) from within the skull. (Choice E) The foramen lacerum is occluded by cartilage. Only small unnamed blood vessels pass through it. (Choice F)The mandibular branch of the trigeminal nerve (CN V3) passes through the foramen ovale.

Educational Objective: The oculomotor nerve (CN Ill), ophthalmic nerve (CN VI) branches, trochlear nerve (CN IV), abducens nerve (CN VI), and superior ophthalmic vein enter the orbit via the superior orbital fissure.

21

Q NO 15: A patient presents to your office complaining of double vision. On neurologic examination, he is unable to adduct his left eye and lacks a corneal reflex on the left side. What is the most likely location of the lesion?

Page 22: Anat 50q.doc

USMLE WORLD STEP 1 ANATOMY

A. AB. BC. CD. DE. EF. F

Explanation:The patient described in the question stem is suffering from symptoms of benign prostatic hyperplasia (BPH). On autopsy, this condition is present in >90% of men aged eighty and above. Urinary complaints are the primary symptoms in BPHI including urinary hesitancy, urgency, frequency, incomplete voiding, post-void leakage of urine, and nocturia. Diagnosis is made from the history and physical, including the digital rectal exam. Medical treatment of BPH is with a-adrenergic blockers, which relax the smooth muscle in the bladder neck, or with 5-α reductase inhibitors, which reduce hormonal influence on the prostate by preventing the conversion of testosterone to dihydrotestosterone. In the pelvic CT “slice” given in the question stem, the prostate is labeled “(A).” At this level of sectioning, the prostate is located posterior to the pubic symphysis and anterior to the anal canal “(B).” (Choice C) C is the distal end of bladder. (Choice D) D labels the ischioanal fossa. (Choice E) E is the rectal abdominus muscle. (Choice F) F is the gluteus maximus muscle.

Educational Objective: BPH is a common, age-related condition that causes urinary symptoms and can be medically treated with a-adrenergic blockers or 5-α reductase inhibitors. The prostate is located between the pubic symphysis and the anal canal in inferior sections of the pelvis on CT scan.

22

Q NO 16: A 63-year-old male complains of having to “strain” on urination. The patient feels much better several weeks after starting doxazosin. Involvement of which of the following structures is most likely responsible for this patient’s symptoms?

Page 23: Anat 50q.doc

USMLE WORLD STEP 1 ANATOMY

A. Gastric antrum B. Duodenal bulb C. Descending portion of the duodenum D. Transverse portion of the duodenum E. Ascending portion of the duodenum F. Duodenojejunal flexure

Explanation:The superior mesenteric artery (SMA) leaves the aorta at the level of Li and supplies the intestine from the duodenum and pancreas to the left colic flexure. The transverse portion of the duodenum lies horizontally at the level of L4, between the aorta and superior mesenteric artery. Normally, the SMA and aorta form, approximately, a 45% angle. If this angle diminishes, the transverse portion of the duodenum can get entrapped between the SMA and aorta, leading to symptoms of partial intestinal obstruction. This condition is called superior mesenteric artery syndrome. Narrowing of the aortomesenteric angle can occur with any condition that causes diminished mesenteric fat or pronounced lordosis, including low body weight or recent weight loss of mesenteric fat (due to burns or other catabolic states) and prolonged bed rest.

Educational Objective: Superior mesenteric artery syndrome occurs when the transverse portion of the duodenum is entrapped between the SMA and aorta causing symptoms of partial intestinal obstruction. This syndrome occurs when the aortomesenteric angle critically decreases either from diminished mesenteric fat or pronounced lordosis.

23

Q NO 17: A 32-year-old Caucasian female presents with epigastric pain, fullness after meals, and bilious vomiting. These symptoms started after she landed a role in a soap opera, inspiring her to lose 25 lbs. on a “crash diet.” On laparotomy, her superior mesenteric artery is observed to take a sharp angle from the aorta. Which of the following structures is most likely to be obstructed by the artery?

Page 24: Anat 50q.doc

USMLE WORLD STEP 1 ANATOMY

A. Bitemporal hemianopsia B. Unilateral deafness C. Ptosis D. Horizontal nystagmus E. Anosmia

Explanation:The star on the chest x-ray above marks the pulmonary apex. Apical lung tumors, or Pancoast tumors (named for the radiologist who first described them), can cause a variety of symptoms due to invasion of nearby structures. Most commonly, Pancoast tumors cause shoulder pain due to invasion of the parietal pleura, vertebrae, and superior ribs. Invasion of the brachial plexus can cause weakness and paresthesias of the arm, particularly at sites innervated by the inferior trunk. Superiorvena cava (SVC) compression by tumor can cause SVC syndrome, and recurrent laryngeal nerve involvement can cause hoarseness. Homer syndrome, the triad of ipsilateral miosis, ptosis and anhidrosis, occurs in up to 50% of patients with Pancoast tumors, and occurs due to tumor invasion of the paravertebral sympathetic chain. (Choice A) Bitemporal hemianopsia classically results from masses within the hypophysial fossa, such as pituitary tumors. Prolactinomas are the most common pituitary tumors. (Choice B) Unilateral deafness can result from many insults including cerumen impaction, damage to the conductive system, or damage to cranial nerve VIII. (Choice D) Horizontal nystagmus can result from disease of the peripheral vestibular system or severe damage to one of the cerebral hemispheres. (Choice E) Anosmia (loss of the sense of smell) is associated with a variety of potential causes including Kallman syndrome, toxin exposure, malignancy invading the olfactory region of the CNS or nasopharynx, etc.

Educational Objective: Apical lung tumors are called Pancoast tumors. Pancoast tumors can cause Homer syndrome, SVC syndrome, arm weakness, arm paresthesias, and hoarseness.

24

Q NO 18: A 56-year-old male smoker is being evaluated for right shoulder pain. You suspect a malignancy in the location marked by a star below. Which of the following additional findings is likely to also be present in this patient as a result of local tumor extension?

Page 25: Anat 50q.doc

USMLE WORLD STEP 1 ANATOMY

A. Biceps brachii B. Supraspinatus C. Levator scapulae D. Serratus anterior E. Deltoid

Explanation:The rotator cuff is made up of the tendons of the following muscles: supraspinatus, infraspinatus, subscapularis, and teres minor. The tendons of these rotator cuff muscles, along with the tendon of the long head of the biceps brachii muscle, and the ligaments of the glenohumeral joint contribute to the stability of the joint between the humerus and the glenoid fossa of the scapula. Of these structures the tendon of the supraspinatus muscle is most commonly affected in rotator cuff syndrome. Due to its superior location this tendon is vulnerable to chronic repeated trauma from impingement between the head of the humerus and the acromioclavicular joint. Motions that typically cause inflammation of this tendon are simultaneous abduction and flexion of the arm at the shoulder but a traumatic fall laterally on an outstretched hand can also cause injury. Inflammation is followed by fibrosis which worsens the problem by increasing friction between the head of the humerus and the acromion, as well as causing inflammation of the subacromial bursa. (Choice A) The tendon of the long head of the biceps brachii muscle passes through the glenohumeral joint to insert on the supraglenoid tubercle of the scapula. The

25

Q NO 19: A 53-year-old Caucasian male presents to your office with a dull ache in his shoulder that interferes with his sleep. Local tenderness is present just below the acromion. Active abduction of the right arm into an overhead position is accompanied by severe pain. A tendon of which of the following muscles is most likely inflamed in this patient?

Page 26: Anat 50q.doc

USMLE WORLD STEP 1 ANATOMY

short head tendon inserts directly onto the anterior tip of the coracoid process of the scapula. (Choice C) The tendons of the levator scapulae insert on the superomedial border of the scapula and on the transverse processes of the C1 through C4 vertebrae. The levator scapulae does not contribute to the stability of the glenohumeral joint. (Choice D) The serratus anterior muscle courses from the medial border of the scapula to insert onto the lateral surfaces of the first eight ribs. Injury to the long thoracic nerve causes paralysis of this muscle and “winging” of the scapula. (Choice E)The deltoid originates on the clavicle, the acromion, and the spine of the scapula as the anterior, medial, and posterior parts of this muscle, respectively. Together, these parts of the deltoid insert by a common tendon on the lateral surface of the humerus.

Educational Objective: The most commonly injured structure in rotator cuff syndrome is the tendon of the supraspinatus muscle. This tendon is vulnerable to injury due to impingement between the acromion and the head of the humerus.

26

Page 27: Anat 50q.doc

USMLE WORLD STEP 1 ANATOMY

A.

Glossopharyngeal nerve B. Vagal nerve C. Chorda tympani D. Maxillary division of the trigeminal nerve E. Mandibular division of the trigeminal nerve

Explanation:Innervation of the tongue is complex, as there are motor, general sensory and gustatory (taste) components. 1. Motor innervation of the tongue is provided by the hypoglossal nerve (ON XII), with the exception of the palatoglossus muscle, which is innervated by the vagus nerve (ON X). 2. General sensory innervation of the tongue (including touch pain, pressure, and temperature sensation) is provided by: • Anterior 2/3 of the tongue: mandibular branch of trigeminal nerve (ON V3) • Posterior 1/3 of the tongue: glossopharyngeal nerve (ON IX) • Posterior area of the tongue root: vagus nerve (ONX) 3. Gustatory innervation (taste buds) is as follows: • Anterior 2/3 of the tongue: chorda tympani branch of facial nerve (ON VII) • Posterior 1/3 of the tongue: glossopharyngeal nerve (ON IX) • Posterior area of the tongue root and taste buds of the larynx and upper esophagus: vagus nerve (ON X) The patient described above has an ulcer on the anterior 2/3 of the tongue (distal to the foramen cecum). Pain from this region (as well as sensations of touch pressure, and temperature) are transmitted by the mandibular branch of the trigeminal nerve.

(Choice A) The glossopharyngeal nerve transmits taste, pain, temperature and touch stimuli from the posterior 1/3 of the tongue. (Choice B) The vagus nerve innervates the far posterior area of the tongue root, transmitting both gustatory and general sensory stimuli. (Choice C) The chorda tympani branch of the facial nerve (CN VII) transmits gustatory sensation from anterior 213 of the tongue but not painful stimuli. (Choice D) The maxillary division of the trigeminal nerve does not participate in tongue innervation.

Educational Objective: General sensation from the anterior 2/3 of the tongue is carried by the mandibular division of the trigeminal nerve. Gustatory innervation of anterior 2/3 of the tongue is provided by chorda tympani branch of the facial nerve.

27

Q NO 20: A 35-year-old HIV-positive male complains of throat pain. Physical examination reveals an ulcer located on the median sulcus of the tongue 1 cm distal to the foramen cecum. The pain sensation is most likely carried by:

Page 28: Anat 50q.doc

USMLE WORLD STEP 1 ANATOMY

A. Pulmonary trunk B. Right ventricle C. Left atrium D. Left ventricle E. Inferior vena cava

Explanation:A penetrating injury at the left sternal border in the fourth intercostal space (level of the nipple) will pass through the following layers:

1. Skin and subcutis 2. Pectoralis major muscle 3. External intercostal membrane 4. Internal intercostal muscle 5. Internal thoracic artery and veins 6. Transversus thoracis muscle 7. Parietal pleura 8. Pericardium 9. Right ventricular myocardium

It is important to know that the right ventricle composes the majority of the anterior surface of the heart. The left lung would not be punctured by a stab wound in this location because there is no middle lobe on the left side and the superior lobe of the left lung is displaced laterally by the cardiac impression. (Choice A) The pulmonary trunk could be pierced by a penetrating injury to the second intercostal space at the left sternal border. (Choice C)The left atrium is located at the base of the heart and makes up most of the heart’s posterior surface. Only the auricle of the left atrium is visible anteriorly (protruding between the pulmonary trunk and the left ventricle). (Choice D) The left ventricle composes the left lateral aspect of the heart. A stab wound in the fourth intercostal space in the midclavicular line could potentially strike the left ventricle, but only after passing through the bulk of the left lung. (Choice E)The inferior vena cava passes through the right side of the central tendon of the diaphragm at the level of T8. A stab wound to the back to the immediate right of the vertebral bodies could strike the IVCI but the injury described above would be unlikely to.

Educational Objective: A penetrating injury at the left sternal border in the fourth intercostal space would puncture the anterior surface of the heart. The right ventricle composes most of the heart’s anterior surface.

28

Q NO 21: A 23-year-old male is rushed to the emergency room after being involved in a street fight. Physical examination reveals a stab wound at the left sternal border in the fourth intercostal space. Which of the following structures is most likely to have been damaged?

Page 29: Anat 50q.doc

USMLE WORLD STEP 1 ANATOMY

A. Pelvic splanchnic nerves B. Pudendal nerves C. Roots of sacral spinal nerves D. Sacral spinal cord E. Thoracic spinal cord

Explanation:This patient is experiencing neurogenic detrusor spasticity, also known as active neuropathic incontinence. The detrusor muscle is the muscle of the urinary bladder wall. It is primarily controlled by local reflex arcs, though there is cortical inhibitory control as well. The reflex arcs include visceral sensory (afferent) neurons that detect bladder wall stretch and visceral motor neurons that stimulate detrusor contraction and expulsion of urine. At rest, this reflex is suppressed by upper motor neurons projecting from supraspinal centers in the pons and cortex, allowing voluntary control of micturition. Spinal cord injury above the conus medullaris can damage these cortical regulatory neurons, causing hyperreflexia in response to stretch (following a brief period of bladder areflexia). Parasympathetic neurons from the pelvic plexus (S2-S4) provide excitatory motor innervation to the detrusor muscle and inhibitory innervation to the internal urethral sphincter. Sympathetic neurons from spinal levels T11-L2 inhibit the detrusor muscle (Choices A, C and D) The pelvic splanchnic nerves, derived from the S2-S4 spinal nerves, mediate parasympathetic innervation to the hindgut and pelvic organs. Pelvic plexus nerves that originate in the sacral micturition center stimulate the detrusor muscle and are responsible for the bladder stretch-contraction reflex arc. Cauda equina lesions that damage the sacral micturition center lead to bladder hypotonia and hyporeflexia and subsequent overflow incontinence. (Choice B) The pudendal nerve originates from the S2-S4 nerve roots and innervates the perineum and genitals. It provides voluntary motor control of the sphincter urethrae and external anal sphincter.

Educational Objective: Damage to the upper motor neurons that control the urinary bladder stretch-contraction reflex arc causes neurogenic detrusor instability. Damage at any above the conus medullaris can cause this problem.

29

Q NO 22: A 53-year-old female complains of urinary frequency. Studies reveal a small bladder that contracts with only minimal stretching. Damage to which of the following structures is likely responsible for this patient’s symptoms?

Page 30: Anat 50q.doc

USMLE WORLD STEP 1 ANATOMY

A. External iliac artery B. Internal iliac artery C. Uterine artery D. Inferior vena cava E. Gonadal vein

Explanation:There are a few important landmarks to keep in mind when asked about the ureters. The ureters originate bilaterally at the renal pelvis and course within the retroperitoneum inferiorly toward the bladder, initially on top of the psoas muscles. Midway from the kidney to the pelvic inlet the gonadal artery and vein cross over the ureter (“water under the bridge”). The ureters then gain access to the pelvis by crossing over (anterior to) the external iliac artery and vein medial to the gonadal vessels and lateral to the internal iliac vessels. The ureters then course through the uterosacral ligament just deep to the uterine vessels in the female before gaining access to the bladder. Due to their course and close proximity to other structures, the ureters are at particular risk of injury during surgery in the pelvis. (Choice A) The ureter courses anterior to the external iliac artery just after the common iliac artery splits into its internal and external branches. After coursing over the external iliac artery, the ureter is considered to be in the pelvis and is medial to the external iliac vessels. (Choice C) Before gaining access to the bladder, the ureter must course within the uterosacral ligament in the female together with the uterine, vaginal, and inferior vesical arteries. The ureter lies medial and deep to the uterine artery with in this ligament. (Choice D) The left ureter lies some distance from the inferior vena cava as the IVC rests on the right half of the vertebral bodies and the abdominal aorta lies on the left half of the vertebral bodies. (Choice E) As mentioned, the gonadal vessels cross over the ureters on both the right and the left with in the abdominal retroperitoneum before the ureters enter the pelvis. In the male, the gonadal vessels never enter the true pelvis: they course around the pelvic brim before entering the deep ring of the inguinal canal. The ureters enter the true pelvis medial to the ovarian vessels in the female.

Educational Objective: The ureters cross over the external iliac vessels and under the gonadal vessels. They pass lateral to the internal iliac vessels and medial to the gonadal vessels as they enter the true pelvis.

30

Q NO 23: A 34-year-old Caucasian female undergoes surgery for a large left-sided adnexal mass. The surgeon can palpate the left ureter immediately lateral to which of the following structures?

Page 31: Anat 50q.doc

USMLE WORLD STEP 1 ANATOMY

A. The right main

bronchus is longer than the left main bronchus B. The right main bronchus is straighter than the left main bronchus C. The right main bronchus is narrower than the left main bronchus D. The right lower lobe receives a richer blood supply than the left lower lobe E. Vascular resistance is higher in the right lower lobe than the right upper lobe

Explanation:The right main bronchus is more prone to foreign body aspiration than the left main bronchus because the right main bronchus has a larger diameter, is shorter, and is oriented more vertically than the left main bronchus. This fact can be remembered using the mnemonic, “Swallow a bite, goes down the right.”

Educational Objective: Aspirated or inhaled particles are most likely to become lodged in the right main bronchus or its branches because this bronchus is shorter, wider and more vertically oriented than the left main bronchus.

31

Q NO 24: A 63-year-old male with a history of stroke and resultant severe oropharyngeal dysphagia develops a right lower lobe pneumonia after an episode of vomiting. This type of aspiration pneumonia commonly affects the right lower lung lobe because:

Page 32: Anat 50q.doc

USMLE WORLD STEP 1 ANATOMY

A. Cochlear base B. Oval window C. Round window D. Helicotrema E. Stapes

Explanation:The inner ear is a fluid-filled site encased in bone that houses the cochlea, the semicircular canals, and the vestibule. The cochlea is a cone-shaped spiraling structure. The base of the cochlea is the widest section of the cone, and it lies near the oval and round windows. The apex of the cochlea is the tip of the cone which lies near the helicotrema. The cochlea is composed of three spiraling fluid-filled ducts known as the scala vestibuli the scala media, and the scala tympani. The scala media lies between the scala vestibuli and the scala tympani. Both the scala vestibuli and the scala tympani are filled with perilymph, which has a high sodium concentration, while the scala media is filled with endolymph having a high potassium concentration. The scala media is separated from the scala tympani by the basilar membrane. The scala media is also unique in that it houses the tectorial membrane and the organ of Corti. The organ of Coil is the specific site where sound is transduced into the nervous system. The transduction of mechanical auditory forces into nervous impulses occurs in the organ of Corti by the following steps. First, sound reaches the middle ear by vibrating the tympanic membrane. This vibration is transferred to the oval window by the ossicles. Vibration of the oval window causes movement of the perilymph in the scala

32

Q NO 25: A 65-year-old male presents to your office complaining of hearing loss. An audiogram shows impaired hearing of low-frequency sounds. Sensory cells close to which of the following structures are most likely affected in this patient?

Page 33: Anat 50q.doc

USMLE WORLD STEP 1 ANATOMY

tympani, and this, in turn, causes the flexible basilar membrane to vibrate. Vibration of the basilar membrane causes bending of the cilia of the hair cells as fluid runs past the rigid tectorial membrane. Hair cell bending causes oscillating hyperpolarization and depolarization, thus creating nervous impulses from sound. At the base of the cochlea, the basilar membrane is thin and rigid and best responds to high frequency sound. The basilar membrane at the apex of the cochlea, near the helicotrema, is large and flexible so it best responds to low frequency sounds. The helicotrema is the site where the scala vestibuli and the scala tympani meet. (Choices A, B and C) The cochlear base lies near the round and oval windows; this segment of the cochlea best responds to high-frequency sound. Older adults do suffer from high-frequency hearing loss much more often than low-frequency loss. The oval window transmits vibration to the scala tympani which communicates with the scala vestibuli at the helicotrema, which then transmits vibration to the round window. As the oval window moves inward, the round window moves outward. (Choice E)The stapes is one of the ossicles of the middle ear. The other two are the malleus and incus.

Educational Objective: Low-frequency sound is best detected at the apex of the cochlea near the helicotrema. High-frequency sound is best detected at the base of the cochlea near the oval and round windows.

33

Page 34: Anat 50q.doc

USMLE WORLD STEP 1 ANATOMY

A. Round ligament B. Suspensory ligament C. Ovarian ligament D. Transverse cervical ligament E. Mesosalpinx

Explanation:The ovaries are the female gonadal organs, located in the pelvis. The ovaries are covered by the mesovarium, which extends to form the broad ligament of the uterus. The uterine tube, ovarian ligament, round ligament of the uterus, and a portion of the suspensory ligament of the ovary are also surrounded by the broad ligament. The nerves, arteries, veins, and lymphatics supplying the ovary are all delivered by the suspensory ligament of the ovary. The ovarian blood supply is provided by the ovarian arteries, which arise from the abdominal aorta bilaterally. The right ovarian vein drains into the inferior vena cava and the left ovarian vein drains into the left renal vein. The parasympathetic innervation of the ovary is derived from the vagus nerve. The ureters course retroperitoneally in close association with the gonadal vessels. A common complication in surgeries ligating the ovarian vasculature in the suspensory ligament is accidental ligation of the ureter. (Choice A) There are two major ‘round ligaments’ in the body: the round ligament of the uterus and the round ligament of the liver. The round ligament of the uterus courses through the inguinal ligament and links the horns of the uterus to the external genitalia. (Choice C) The ovarian ligament carries no named blood vessels. It courses from the uterine pole of the ovary to the body of the uterus just below the uterine tubes. (Choice D) The transverse cervical ligament, also known as the cardinal ligament, extends from the cervix and lateral fornix of the vagina to the lateral pelvic walls. The uterine artery courses in this tissue and requires ligation during radical hysterectomy. (Choice E)The mesosalpinx is the region of the broad ligament of the uterus that lies between the uterine tube and the ovary.

Educational Objective: The nerves and vessels supplying the ovary are delivered through the suspensory ligament of the ovary.

34

Q NO 26: A 54-year-old female with chronic pelvic pain is found to have a right-sided ovarian mass. After the initial evaluation, surgery is planned to remove the mass. To avoid excessive bleeding during the surgery the surgeon should ligate which of the following structures?

Page 35: Anat 50q.doc

USMLE WORLD STEP 1 ANATOMY

A. Carpal tunnel B. Hook of the hamate C. Surgical neck of the humerus D. Head of the radius E. Coracobrachialis muscle

Explanation:The ulnar nerve is a branch of the medial cord of the brachial plexus derived from the C7-T1 ventral rami. The ulnar nerve provides sensory innervation to the fifth digit and the medial half of the fourth digit as well as to the palmar and dorsal surfaces of the hand. The ulnar nerve also provides motor innervation to the flexor carpi ulnaris and to the medial section of the flexor digitorum profundis in the forearm. In the hand, the ulnar nerve innervates all of the palmar and dorsal interosseus muscles the muscles of the hypothenar eminence, the palmaris brevis muscle, the fourth and fifth lumbrical muscles, and the adductor pollicis muscle. The ulnar nerve enters the forearm after passing behind the medial epicondyle of the humerus where it is covered by a small amount of overlying soft tissue. This region, sometimes referred to as the “funny bone1” is a common site of ulnar nerve injury’. Ulnar nerve injury’ at this site classically causes a “claw hand” deformity. In the wrist, the ulnar nerve passes between the hook of the hamate and the pisiform bone in a fibroosseous tunnel known as Guyon’s canal. Ulnar nerve injury’ at Guyon’s canal causes dysesthesia of the ulnar side of the hand and weakness of the intrinsic muscles of the hand. (Choice A) In carpal tunnel syndrome, some factor reduces the size of the carpal tunnel causing median nerve compression. Patients typically experience difficulty’ with fine motor control of the thumb. (Choice C) Fracture of the surgical neck of the humerus may cause axillary’ nerve injury’ leading to paralysis of the deltoid and teres minor muscles as well as loss of sensation of the lateral upper arm. (Choice D) The deep branch of the radial nerve can be affected by radial head subluxation leading to weakness or paralysis of many of the muscles of the extensor compartment of the forearm. (Choice E) The coracobrachialis muscle is an arm flexor that lies deep to the biceps brachii and overlies the median nerve and brachial artery. It is innervated by the musculocutaneous nerve.

Educational Objective: Ulnar nerve injury classically causes a “claw hand” deformity. The ulnar nerve can be injured either near the medial epicondyle of the humerus or in Guyon’s canal near the hook of the hamate and pisiform bone in the wrist.

35

Q NO 27: A 19-year-old female presents to your office complaining of “right hand clumsiness.” Physical examination reveals decreased sensation over the fifth finger and a flattened hypothenar eminence. The affected nerve is commonly injured at which of the following locations?

Page 36: Anat 50q.doc

USMLE WORLD STEP 1 ANATOMY

A. Anterior thigh B. Medial leg C. Medial foot D. Dorsal foot E. Plantar foot

Explanation:The tibial nerve is the large medial branch of the sciatic nerve that descends through the popliteal fossa together with the popliteal vein and artery. Injury to this nerve can occur after deep penetrating trauma to the popliteal fossa and can cause a number of different deficits due to its numerous functions. The tibial nerve innervates the gastrocnemius, soleus and plantaris muscles which are responsible for plantarflexion of the foot. It also supplies the flexor digitorum longus and flexor hallucis longus, which are responsible for toe flexion. The tibial nerve also innervates the tibialis posterior muscle which is responsible for inversion of the foot. (Patients with injury to the tibial nerve often hold their lower extremity in a calcaneovalgocavus position—the foot is dorsiflexed and everted.) After delivering its motor innervation, the tibial nerve terminates by dividing into the medial and lateral plantar nerves to provide sensory innervation to the skin of the distal plantar surface of the foot. (Choice A) The cutaneous branch of the femoral nerve innervates the majority of the skin of the anterior thigh. (Choice B) Cutaneous branches of the saphenous nerve innervate the skin of the medial leg. The saphenous nerve is a branch of the femoral nerve and courses to the medial leg together with the great saphenous vein. (Choice C)The medial aspect of the foot is innervated by the superficial peroneal nerve, a branch of the common peroneal nerve, as well as by the saphenous nerve.

36

Q NO 28: A 34-year-old male has difficulty walking after sustaining a traumatic injury to his right leg. Physical examination reveals a right foot that is dorsiflexed and everted. The patient is unable to stand on his tiptoes. What is the most likely area of sensory loss in this patient?

Page 37: Anat 50q.doc

USMLE WORLD STEP 1 ANATOMY

(Choice D)The superficial peroneal nerve innervates the dorsum of the foot except for the skin between the great toe and the second toe, which is innervated by the deep peroneal nerve.

Educational Objective: The tibial nerve innervates the flexors of the lower leg, the extrinsic digital flexors of the toes, and the skin of the sole of the foot.

37

Page 38: Anat 50q.doc

USMLE WORLD STEP 1 ANATOMY

A. Taste sensation from the anterior two-thirds of the tongue B. General sensation from the tonsillar lining C. Salivary secretion from the submandibular gland D. Protrusion of the tongue

Explanation:The glossopharyngeal nerve (CN IX) originates in the medulla and exits the cranial cavity via the jugular foramen. This nerve has numerous functions: Somatic motor stylopharyngeus muscle only Parasympathetic: inferior salivatory nucleus—CN IX—otic ganglion— travels along auriculotemporal nerve (CN V)—parotid gland secretion.General sensory inner surface of tympanic membrane, Eustachian tube, posterior 1/3 of tongue, tonsillar region, and upper pharynx(afferent portion of the gag reflex), carotid body and carotid sinus Lesions of the glossopharyngeal nerve result in loss of the gag reflex (afferent limb), loss of sensation in the upper pharynx, posterior tongue tonsils and middle ear cavity, and loss of taste sensation on the posterior one-third of the tongue. (Choice A) Taste sensation from the anterior two-thirds of the tongue is mediated by the chorda tympani branch of the facial nerve (CN VII). (Choice C) Salivary secretion from the submandibular gland and the sublingual gland is mediated by parasympathetic fibers originating in the superior salivatory nucleus carried on the facial nerve (CN VII) via the chorda tympani nerve, the lingual nerve, and across the submandibular ganglion. (Choice D) Protrusion of the tongue is mediated by motor efferent fibers carried by the hypoglossal nerve (CN XII).

Educational Objective: Lesions of the glossopharyngeal nerve result in loss of the gag reflex (afferent limb), loss of sensation in the upper pharynx, posterior tongue tonsils, and middle ear cavity, and loss of taste sensation on the posterior one-third of the tongue.

38

Q NO 29: The glossopharyngeal nerve is transected accidentally during a surgical procedure in a 45-year-old male. Which of the following is most likely lost in this patient?

Page 39: Anat 50q.doc

USMLE WORLD STEP 1 ANATOMY

A.

Anterior cerebral artery B. Middle cerebral artery C. Posterior cerebral artery D. Anterior inferior cerebellar artery E. Posterior inferior cerebellar artery

Explanation:This patient has fluent aphasia, which is a form of receptive aphasia characterized by speech that is fluent, well articulated, and melodic — but meaningless. Individuals stricken with fluent aphasia cannot understand verbal or written language, whether it is spoken to them or by them. Typically, there is no awareness of this cognitive deficit on the part of the individual. Fluent aphasia is often produced by a lesion in Wernicke’s area, which is located in the auditory association cortex of the posterosuperior first temporal gyrus in the dominant temporal lobe. Because the left hemisphere is dominant in most individuals, the left temporal lobe is the typical site of Wernicke’s area. The mnemonic used to remember the association between Wernicke’s area and fluent aphasia is “Wernicke’s aphasia causes word salad.” The regions of the cerebral cortex supplied by the middle cerebral artery (MCA) are approximated by the red oval on the diagram below. The inferior terminal MCA branches supply Wernicke’s area.

(Choices A, C, D, and E) The anterior cerebral artery, posterior cerebral artery, anterior inferior cerebellar artery, and posterior inferior cerebellar artery do not typically service Wernicke’s area. Consequently, occlusions of these arteries would not be associated with Wernicke’s aphasia.

Educational Objective: A lesion in Wernicke’s area can cause receptive aphasia, which is characterized by well-articulated, nonsensical speech paired with a lack of language comprehension (“Wernicke’s aphasia causes word salad”). Wernicke’s area is located in the auditory association cortex of the posterosuperior first temporal gyrus in the dominant temporal lobe and receives its blood supply from the inferior branches of the middle cerebral artery.

39

Q NO 30: A highly agitated 54-year-old male is brought to the emergency department. He speaks clearly and with conviction but does not appear to understand the doctor’s questions. He does not follow oral or written instructions and cannot repeat simple phrases. Occlusion of which of the following arteries is most likely responsible for this patient’s condition?

Page 40: Anat 50q.doc

USMLE WORLD STEP 1 ANATOMY

A. Anterior circumflex humeral artery B. Brachial artery C. Deep brachial artery D. Radial collateral artery E. Common interosseous artery

40

Q NO 31: A 24-year-old mountain climber presents to the emergency room with right shoulder pain following a fall. Physical examination reveals total inability to extend the right wrist. X-ray reveals a fracture of the right humeral shaft. Which of the following arteries is likely to be injured in this patient?

Page 41: Anat 50q.doc

USMLE WORLD STEP 1 ANATOMY

Explanation:When the midshaft of the humerus is fractured, there is significant associated risk of injury to the radial nerve and deep brachial artery. The radial nerve innervates most of the forearm flexors at the elbow and most of the hand flexors at the wrist. It also innervates the extrinsic extensors of the digits, and the brachioradialis and supinator muscles. The radial nerve also provides cutaneous sensory innervation to the dorsal hand, forearm and upper arm. The deep brachial artery branches oft of the brachial artery high in the arm, passes inferior to the teres major muscle, and courses posteriorly along the humerus in close association with the radial nerve. The patient described above is exhibiting signs of radial nerve injury. The combination of radial nerve deficits with radiographic evidence of a humeral fracture should raise concern for an associated injury to the deep brachial artery. (Choice A) The anterior circumflex humeral artery is a branch of the axillary artery that passes anterior to the surgical neck of the humerus and anastomoses with the posterior circumflex humeral artery. A fracture to the surgical neck of the humerus may damage this vessel and the axillary nerve. (Choice B) The axillary artery gives off the posterior circumflex humeral artery and becomes the brachial artery. The brachial artery courses anteromedially in the arm within the bicipital groove and branches into the radial and ulnar arteries after entering the forearm. Supracondylar fractures of the humerus may injure this artery. (Choice D)The deep brachial artery divides into the radial collateral artery and middle collateral artery at the midpoint of the humerus. The radial collateral artery courses with the radial nerve. (Choice E)The common interosseous artery is a short branch of the ulnar artery that gives rise to anterior, posterior and recurrent branches within the proximal forearm.

Educational Objective: The deep brachial artery and radial nerve course along the posterior aspect of the humerus. Fractures of the humerus midshaft risk injury to these structures. Supracondylar fractures are associated with injury to the brachial artery.

41

Page 42: Anat 50q.doc

USMLE WORLD STEP 1 ANATOMY

Which of the following most likely prevented the proper ascent of the anomalous organ shown above?

A. Superior mesenteric artery B. Vitellointestinal duct C. Persistent urachus D. Inferior vena cava E. Inferior mesenteric artery

42

Q NO 32: A patient is being evaluated for recurrent urinary tract infections. An intravenous pyelogram of the patient is shown below.

Page 43: Anat 50q.doc

USMLE WORLD STEP 1 ANATOMY

Explanation:This intravenous pyelogram shows the kidneys joined at their lower poles. This is the most common variant of horseshoe kidney (90%). In the other 10% of cases the fusion occurs at the upper poles. Horseshoe kidney is found in approximately out of every 500 autopsies, and may lead to urinary tract obstruction and the development of hydronephrosis (such as shown on this radiograph). During embryogenesis, the kidney goes through the stages of pronephros, mesonephros and metanephros. Fetal metanephros is initially located in the sacral region, while in the adult the mature kidney is located at vertebral levels T12-L3 (remember that the kidney is slightly lower on the right compared to the left). This relative ascent of the kidneys from the sacral region to their normal anatomic position results from the disproportionately rapid growth of the caudal part of the embryo. When fusion of the lower or upper poles of the kidney occurs, the central part of the newly formed horseshoe kidney lies across the midline anterior to the great vessels. This centrally located isthmus gets trapped by the inferior mesenteric arte during the ascent of the kidney. (Choice A) Superior mesenteric artery is located higher than inferior mesenteric artery. It leaves the aorta at the level of L1 and doesn’t serve as an obstacle for the ascent of a horseshoe kidney. (Choice B) When vitelline duct persists, the connection between the intestinal lumen and the outside of the body is formed at the umbilicus. The discharge of meconium from the umbilicus will be noted in this condition. (Choice C) A direct connection between the bladder lumen and the outside of the body at the umbilicus is called persistent urachus or urachal fistula. This condition would likely have been identified earlier in life. (Choice D)The inferior vena cava lies behind the isthmus of a horseshoe kidney and would not obstruct its ascent.

Educational Objective: In horseshoe kidney, both kidneys are fused together at the poles in early embryonic life. The isthmus of horseshoe kidney usually lies anterior to the aorta and inferior vena cava and posterior to the inferior mesenteric artery. This centrally located isthmus gets trapped behind the inferior mesenteric artery during the relative ascent of the kidney.

43

Page 44: Anat 50q.doc

USMLE WORLD STEP 1 ANATOMY

A. L2 B. L3 C. L4 D. L5 E. S1

Explanation:The patient described in the question stem is suffering symptoms consistent with sciatic neuropathy, or sciatica. Sciatic neuropathy can result from any irritation of the sciatic nerve or its roots. Common causes include vertebral disc herniations, spinal canal stenosis, irritation by bone spurs, and irritation by the piriformis. Classic symptoms of sciatica include weakness of the posterior thigh muscles as well as weakness of all muscles below the knee. The sciatic nerve also provides sensory innervation to the posterior thigh and calf as well as the plantar surface of the foot. Transection of the sciatic nerve would result in an inability to flex the leg at the knee and paralysis of the muscles below the knee. The ankle reflex is diminished in sciatic nerve injury but the patellar reflex is unaffected as this reflex arc is carried by fibers of the femoral nerve (L2—L4). The sciatic nerve is derived from the L4 through 53 nerve roots, so only choices C, D, and E are reasonable options. Sciatica most commonly involves L5 or S1. Of these, compression of the L5 root typically results in posterior and lateral thigh and leg pain shooting to the inner foot. Compression of the S1 root results in pain strictly in the posterior thigh and leg shooting to the foot accompanied by weakness of plantarflexion and loss of the ankle jerk reflex. (Choices A and B) L2 and L3 contribute to the lateral femoral cutaneous nerve as well as other nerves such as the femoral, obturator and genitofemoral. The lateral femoral cutaneous is a purely sensory nerve that enters the thigh deep to the inguinal ligament and supplies most of the skin of the anterior thigh. (Choice C) L4 contributes partially to the femoral, sciatic, obturator, superior gluteal, and quadratus femoris nerves. This nerve provides no sensation to the posterior thigh or leg. Damage to L4 would result in impairment of the knee jerk reflex.

Educational Objective: Sciatica is a painful condition characterized by shooting pain down the posterior thigh and leg that typically results from impingement of one of the spinal nerves as it leaves the vertebral column. Compression of the S1 root results specifically in pain purely in the posterior thigh and leg as well as diminution of the ankle jerk reflex.

44

Q NO 33: A 43-year-old Caucasian female presents to your office with back pain after dragging a heavy box. She describes pain in the right posterior thigh and posterior calf shooting down to the foot. Physical examination reveals symmetric knee reflexes, but the ankle jerk reflex is slightly diminished on the right. Which of the following nerve roots is most likely affected in this patient?

Page 45: Anat 50q.doc

USMLE WORLD STEP 1 ANATOMY

A. Superior laryngeal nerve B. Recurrent laryngeal nerve C. Ansa cervicalis D. Hypoglossal nerve E. Accessory nerve

Explanation:The inferior thyroid artery arises from the thyrocervical trunk, a branch of the subclavian artery. Along with the superior thyroid arteries (branches of the external carotid arteries)the inferior thyroid arteries provide blood to the thyroid gland and other structures of the anterior neck. From the thyrocervical trunk the inferior thyroid artery courses deep to the vagus nerve, common carotid artery and internal jugular vein before turning toward the thyroid. Just before meeting the thyroid gland, this artery courses just superficial to the recurrent laryngeal nerve. The recurrent laryngeal nerve is a branch of the vagus nerve that loops below the aortic arch on the left and below the subclavian artery on the right to provide motor innervation to the muscles of the larynx (all except the cricothyroid muscle). Unilateral injury to this nerve causes hoarseness; bilateral injury causes respiratory difficulty due to airway obstruction by immobile vocal folds. (Choice A) The superior laryngeal nerve is a branch of the vagus nerve that gives oft internal and external branches. The superior thyroid artery courses with the external branch of this nerve; ligation of this artery during surgery may damage the external laryngeal nerve. (Choice C) The ansa cervicalis is a loop of the cervical plexus that receives contributions from C1 C2 and C3. It courses deep to the sternocleidomastoid muscle and loops around the internal jugular vein. Branches from the ansa cervicalis innervate the sternohyoid, sternothyroid and omohyoid muscles. (Choice D) The hypoglossal nerve (CN XII) exits the cranium through the hypoglossal canal and innervates all of the intrinsic muscles of the tongue except the palatoglossus. (Choice E) The spinal accessory nerve (CN XI) exits the jugular foramen and courses on top of the levator scapulae muscle to innervate the sternocleidomastoid and trapezius muscles on either side.

Educational Objective: The recurrent laryngeal nerve travels in close approximation to the inferior thyroid artery and can be injured in surgical procedures of the anterior neck (e.g. thyroidectomy), resulting in laryngeal muscle paralysis, hoarseness and dyspnea.

45

Q NO 34: A 34-year-old male with a squamous cell carcinoma undergoes surgical neck dissection. While attempting to ligate the inferior thyroid artery the surgeon accidentally damages a nerve that lies in close proximity to it. Which of the following nerves was most likely damaged?

Page 46: Anat 50q.doc

USMLE WORLD STEP 1 ANATOMY

A. Axillary nerve B. Radial nerve C. Musculocutaneous nerve D. Long thoracic nerve E. Brachial plexus, lower trunk

Explanation:Of the choices listed, only injury to the lower trunk of the brachial plexus would cause hand weakness. Sudden upward jerking of the arm at the shoulder, as occurred in this patient, can cause injury to the lower trunk of the brachial plexus. The lower trunk of the brachial plexus carries nerve fibers from the CS and Ti spinal levels that ultimately contribute to the median and ulnar nerves. Together these nerves innervate all of the intrinsic muscles of the hand. Thus injury to the lower trunk of the brachial plexus can cause hand clumsiness or paralysis. (Choice A) Shoulder dislocations or fractures of the proximal humerus can cause axillary nerve injury. Weakness of the deltoid and teres minor muscles may result. (Choice B) Fractures at the midshaft of the humerus commonly cause radial nerve injury. Patients may experience posterior arm numbness and paralysis of the arm and forearm extensors.

46

Q NO 35: A 12-year-old boy falls from a tree he is climbing, but catches himself on a branch with his right hand. He swings by his right arm and jumps to the ground. Several hours later he presents to the emergency room with right hand clumsiness. Which of the following structures has he most likely injured?

Page 47: Anat 50q.doc

USMLE WORLD STEP 1 ANATOMY

(Choice C) The musculocutaneous nerve may be damaged in injuries involving the upper trunk of the brachial plexus. The upper trunk of the brachial plexus is classically injured when the head and shoulder are violently stretched apart. The musculocutaneous nerve innervates the biceps brachii and brachialis muscles, so injury to this nerve would cause weakness of forearm flexion at the elbow. (Choice D) Long thoracic nerve injury classically occurs during axillary lymph node dissection and results in paralysis of the serratus anterior muscle leading to scapular” winging” on physical examination.

Educational Objective: Sudden upward stretching on the arm at the shoulder can damage the lower trunk of the brachial plexus. This trunk carries nerves from the CS and Ti spinal levels that ultimately form the median and ulnar nerves. These nerves innervate all of the intrinsic muscles of the hand.

47

Page 48: Anat 50q.doc

USMLE WORLD STEP 1 ANATOMY

A. Suprascapular nerve B. Long thoracic nerve C. Axillary nerve D. Accessory nerve E. Radial nerve

Explanation:This patient is suffering from a pressure-induced radial nerve injury, most likely caused by improperly fitted crutches. The radial nerve is derived from the C5-T1 spinal nerves, and is one of the two final branches of the posterior cord of the brachial plexus. The radial nerve initially courses medial to the surgical neck of the humerus inferior to the teres major muscle within the axilla before entering the posterior arm to course between the long head of the triceps brachii and the posterior humerus. The radial nerve can be injured at its superficial location within the axilla by repetitive pressure and trauma caused by an ill-fitting crutch (“crutch palsy”). The radial nerve and deep brachial artery can also be injured by midshaft humerus fractures. Injury to the radial nerve at either location causes weakness or paralysis of the extensor muscles of the forearm and wrist as well as to the extrinsic extensors of the digits, a motor deficit described clinically as “wrist drop.” (Choice A) The suprascapular nerve innervates the supraspinatus and infraspinatus muscles. These muscles function to abduct and laterally rotate the arm, respectively.

48

Q NO 36: A 25-year-old male fractures his right tibia in a motor vehicle accident. His right leg is fixed in a cast, and he requires crutches to ambulate. Two weeks later, he presents with difficulty extending his arm. Injury to which of the following nerves is most likely responsible?

Page 49: Anat 50q.doc

USMLE WORLD STEP 1 ANATOMY

(Choice B) Damage to the long thoracic nerve causes paralysis of the serratus anterior muscle and winging of the scapula. This nerve is often injured during lymph node dissection in patients undergoing radical mastectomy. (Choice C) Fracture of the surgical neck of the humerus and anterior dislocation of the glenohumeral joint can cause injury to the axillary nerve. The resultant paralysis of the deltoid and teres minor muscles causes weakness of abduction extension flexion and lateral rotation of the arm at the shoulder. (Choice D)The spinal accessory nerve (CN XI) exits the jugular foramen and courses over the levator scapulae muscle to innervate the sternocleidomastoid and trapezius muscles bilaterally.

Educational Objective: Improperly fitted crutches can cause radial nerve injury resulting in weakness of all forearm, wrist and finger flexors (“Wristdrop”).

49

Page 50: Anat 50q.doc

USMLE WORLD STEP 1 ANATOMY

A. Styloid process B. Lesser horn of the hyoid C. Stapes D. Malleus E. Thyroid cartilage

Explanation:The infant described in the question is suffering a defect in development of the first pharyngeal (branchial) arch. There are six branchial arches in the developing embryo, and all but the fifth arch contributing structures to the adult; the fifth arch regresses completely. Each pharyngeal arch is associated with a cranial nerve, and knowledge of which nerve is paired with which arch can assist in remembering which adult structures are derived from each arch. In addition, note that the muscular elements are formed by mesoderm and the bony elements are formed by neural crest cells that migrate in. The first arch is associated with the trigeminal nerve (CN V). The neural crest cells of the first arch form the bones associated with this structure, which include the maxilla, zygoma, mandible vomer, palatine, incus, and malleus (Choice D). Aberrant neural crest cell migration into the first arch can cause bony deformities as described in the question stem. Mesodermal derivatives include all muscles of mastication, anterior belly of the digastric, mylohyoid, tensor tympani and tensor veli palatini. (Choices A, B and C) The styloid process, lesser horn of the hyoid, and the stapes are all neural crest derivatives of the second pharyngeal arch. The second arch is associated with the facial nerve (CN VII) and gives rise to the muscles of facial expression the styloid, the stapedius, and the posterior belly of the digastric. (Choice E) The fourth and sixth arches together make up the cartilaginous structures of the larynx.

Educational Objective: The first arch can be poorly formed during embryonic development resulting in first arch syndrome. Abnormalities include malformation of the mandible, maxilla, malleus, incus, zygoma, vomer, palate, and temporal bone. The first arch is associated with the trigeminal nerve.

50

Q NO 37: An infant born to a 22-year-old female has difficulty feeding because of an underdeveloped mandible. Development of which of the following structures is likely also impaired in this patient?

Page 51: Anat 50q.doc

USMLE WORLD STEP 1 ANATOMY

A. Right internal iliac vein B. Inferior mesenteric vein C. Right renal vein D. Hemiazygous vein E. Inferior vena cava

Explanation:This patient has a varicocele of the right testicle, as indicated by the ‘bag of worms’ texture of the scrotum on the affected side. This texture is produced by the reticular network of veins that drain the testicles the pampiniform plexus. Failure of the venous valves leads to pooling of blood in the testicles due to their dependent location causing varicoceles. Varicoceles can also result from upstream compression of the veins that drain the testes by abdominal or pelvic masses or other forces. Recall that the venous drainage of the testes is not bilaterally symmetric. The right testicular vein drains in to the inferior vena cava (IVC) whereas the left testicular vein drains into the left renal vein which drains into the IVC. The left testicle is more commonly affected by varicocele, perhaps because the left renal vein passes between the aorta and the superior mesenteric artery(SMA), and enlargement or hardening of the SMA may cause compression of the left renal vein and pooling of blood in the left testicle. (Choice A) The right internal iliac vein receives drainage from the pelvic viscera and genitalia (other than the gonads) and the gluteal region and regions served by the pudendal circulation. (Choice B) The inferior mesenteric vein drains the structures of the hindgut. (Choice C) The right renal vein drains only the right kidney. The right suprarenal gland drains directly into the IVC as opposed to the left suprarenal gland which drains primarily into the left renal vein. (Choice D)The hemiazygous vein drains blood from the posterior walls of the thorax and abdomen. Together with the azygous vein and accessory hemiazygous vein, the hemiazygous vein provides an alternate path way for blood to return to the heart bypassing the IVC.

Educational Objective: The right testicular vein drains directly into the IVC while the left testicular vein drains into the left renal vein.

51

Q NO 38: A 24-year-old male presents to your office complaining of lower abdominal discomfort. Physical examination reveals right-sided testicular enlargement that feels like a ‘bag of worms when the patient stands up. The venous blood from the affected testicle drains directly into the:

Page 52: Anat 50q.doc

USMLE WORLD STEP 1 ANATOMY

A. Phrenic B. Accessory C. Vagus D. Superior epigastric E. Long thoracic

Explanation:Pleuritic chest pain can result from any condition that causes inflammation of the pleura. The pleura is divided into segments, as follows: 1. Visceral pleura: The visceral pleura, or pulmonary pleura, cover all surfaces of the lungs, including the surfaces within the pulmonary fissures. 2. Parietal pleura: This represents the remainder of the pleura that is not in contact with the lungs and can be subdivided as follows: • Costal pleura: Covers the thoracic wall including the ribs, sternum, intercostal spaces, costal cartilages, and the sides of the thoracic vertebrae. • Mediastinal pleura: Covers the mediastinum • Diaphragmatic pleura: Covers the surface of the diaphragm located within the thoracic cavity • Cervical pleura: Extends with the apices of the lung into the neck. The parietal pleura is innervated by somatic sensory (sensory afferent) nerves which allow the sensation of sharp and localized pain. The phrenic nerve, which is derived from the C3-C5 nerve roots, delivers motor innervation to the diaphragm and additionally carries pain fibers from the diaphragmatic and mediastinal pleura. Irritation of the pleura in either of these areas will cause a sharp pain worsened by inspiration that will be “referred” to the C3-C5 distribution, which lies at the base of the neck and over the shoulder. Sensory innervation of the remainder of the parietal pleura is accomplished by intercostal nerves. (Choice B) The spinal accessory nerve is the 11th cranial nerve and provides motor innervation to the sternocleidomastoid and trapezius muscles. (Choice C) The vagus nerve is the 10th cranial nerve and is the major source of parasympathetic innervation to the viscera of the chest wall and the foregut. (Choice D)There is no superior epigastric nerve. The superior epigastric artery and vein course over the superior half of the abdominal wall and anastomose with the internal thoracics and the inferior epigastrics. (Choice E) The long thoracic nerve innervates the serratus anterior. Damage to this nerve causes winged scapula.

Educational Objective: Irritation of the mediastinal or diaphragmatic parietal pleura will cause sharp pain, worse on inspiration, in the C3-C5 distribution. Pain sensation from these areas is carried by the phrenic nerve.

52

Q NO 39: A 26-year-old male presents to the ER with a sharp pain in his neck and shoulder. He has some breathing difficulty secondary to pain with inspiration. His past medical history is not significant except for a mild respiratory disease a few days ago. His blood pressure is 120/70 mmHg and his heart rate is 100/mm. The pain experienced by this patient is most likely carried by which of the following nerves?

Page 53: Anat 50q.doc

USMLE WORLD STEP 1 ANATOMY

A. Sternoclavicular ligament B. Costoclavicular ligament C. Acromioclavicular ligament D. Coracoclavicular ligament F. Levator scapulae muscle

Explanation:The clavicle is anchored laterally to the scapula at the shoulder by two major ligaments: the acromioclavicular ligament superiorly and the coracoclavicular ligament inferiorly. The acromioclavicular ligament provides the strongest support between these structures and overlies a synovial joint where the clavicle and acromion meet (acromioclavicular joint, or AC joint). The coracoclavicular ligament is actually composed of two distinct bands of ligamentous tissue known as the trapezoid ligament and the conoid ligament. As the name implies, this ligament binds the coracoid process of the scapula to the clavicle. Trauma to the shoulder can separate the scapula and clavicle. This injury is known as acromioclavicular subluxation, or shoulder separation, and is a unique entity from shoulder dislocation. In this type of injury, either ligament, or both, can rupture. If both ligaments are torn, the arm and scapula fall away from the clavicle from the force of gravity, making the clavicle quite prominent on physical examination. (Choices A and B) The anterior and posterior sternoclavicular ligaments and costoclavicular ligaments anchor the clavicle to the sternum and ribs in the midline. There is also an interclavicular ligament in this region that binds the medial ends of each clavicle to one another. (Choice E)The levator scapulae muscle attaches to the transverse processes of the C1-C4 vertebrae and the superomedial border of the scapula. It functions to elevate and rotate the scapula and has no function in supporting the shoulder or AC joint.

Educational Objective: The scapula and clavicle are firmly bound together distally and superiorly by the acromioclavicular ligament and joint: they are bound more proximally and inferiorly by the coracoclavicular ligament. Rupture of these ligaments can occur with trauma to the shoulder, which will cause the arm and scapula to fall away from the clavicle.

53

Q NO 40: An 11-year-old boy presents to the ER after a basketball injury caused acute right shoulder pain. Imaging studies suggest clavicular dislocation at the acromioclavicular joint. Which of the following acts to prevent the dislocation described in this patient?

Page 54: Anat 50q.doc

USMLE WORLD STEP 1 ANATOMY

A. Thyroarytenoid B. Lateral cricoarytenoid C. Posterior cricoarytenoid D. Cricothyroid E. Aryepiglotticus

Explanation:The superior thyroid artery, a branch of the common carotid artery, and the inferior thyroid artery, a branch of the thyrocervical trunk, provide the blood supply to the thyroid and parathyroid glands. The superior thyroid artery, superior thyroid vein and external branch of the superior laryngeal nerve course together in a neurovascular triad that originates superior to the thyroid gland and lateral to the thyroid cartilage. Because the external branch of the superior laryngeal nerve courses just deep to the superior thyroid artery, it is at risk of injury during thyroidectomy. The cricothyroid muscle is the only muscle innervated by his nerve. The remaining laryngeal muscles are innervated by the recurrent laryngeal nerves, which also provide sensory innervation to the larynx below the vocal folds. The internal branch of the superior laryngeal nerve does not innervate any muscles, but provides sensory innervation to the laryngeal mucosa above the vocal folds. (Choices A, B, C and E)These muscles are all innervated by the recurrent laryngeal nerve.

Educational Objective: The external branch of the superior laryngeal nerve is at risk of injury during thyroidectomy due to its proximity to the superior thyroid artery and vein. This nerve innervates the cricothyroid muscle.

54

Q NO 41: A 54-year-old female undergoes thyroidectomy for several thyroid nodules that were suspicious for malignancy on fine needle aspiration. While ligating the superior thyroid artery, the surgeon accidentally severs a nearby nerve. Which of the following muscles is denervated as a result of this injury?

Page 55: Anat 50q.doc

USMLE WORLD STEP 1 ANATOMY

A. Mesolimbic-mesocortical B. Nigrostriatal C. Tuberoinfundibular D. Periventricular E. Incerthypothalamic

Explanation:The patient appears to be suffering from galactorrhea, a non-puerperal secretion of watery or milky fluid that contains neither pus nor blood. The tuberoinfundibular dopaminergic pathway connects the hypothalamus and the pituita gland and is responsible for dopamine-dependent prolactin tonic inhibition. The neurotransmitter dopamine is secreted from these neural axons and inhibits prolactin secretion from the posterior pituita gland. Some antipsychotic drugs block dopamine in the tuberoinfundibular pathway, which can cause an increase in blood prolactin levels, leading to hyperprolactinemia and abnormal lactation (galactorrhea), disruptions to the menstrual cycle in women, and sexual dysfunction. (Choice A) The mesolimbic-mesocortical pathway is primarily involved in regulating behavior. This area is hyperactive in schizophrenia. Antipsychotic action of the neuroleptics is by primarily blocking the dopamine in the mesolimbic-mesocortical (frontal cortex) pathway. (Choice B) In the nigrostriatal system the neurons are projected from the substancia nigra to the caudate nucleus and putamen. This pathway primarily regulates coordination of voluntary movements. In this pathway dopamine inhibits the release of acetylcholine. Degeneration of the substantia nigra thus causes decreased dopamine and subsequent increased acetylcholine; this leads to hyperkinetic disorders such as Parkinsonism. Administering high potency antipsychotics can cause some dopamine antagonism in this pathway resulting in drug-induced Parkinsonism. (Choices D and E)The other 2 pathways are not really well defined and not important for purpose of board examination.

Educational Objective: Remember the 3 important Dopaminergic systems:

55

Q NO 42: A 26-year-old female presents to your office with a three-month hi stow of amenorrhea. She also notes that during this time her breasts have become engorged. She is taking a drug that helps her ‘not to hear voices’ and acetaminophen for occasional headaches. You proceed with giving her a pregnancy test that returns with a negative result. Which of the following dopaminergic pathway disturbances are most likely responsible for this patient’s symptoms?

Page 56: Anat 50q.doc

USMLE WORLD STEP 1 ANATOMY

Explanation:The patient described in the question stem is experiencing a very common adverse effect of multiple sclerosis: demyelinization of the optic nerve (CN II). The examiner in the question stem is testing the light reflex pathway. The optic nerve carries the afferent (sensory) limb of this pathway. The efferent limb (motor) is carried by the oculomotor nerve (CN Ill). In brief light enters the eye and generates a signal that is

56

Q NO 43: A 23-year-old Caucasian female suffers from an acute attack of multiple sclerosis. Impulse transmission along the left optic nerve is hindered by an area of demyelinization. Which of the following is the most likely finding during the physical exam?

Page 57: Anat 50q.doc

USMLE WORLD STEP 1 ANATOMY

transmitted from the retina along the optic nerve to synapse in the ipsilateral pretectal nucleus located in the superior colliculus. Fibers then project from the pretectal nucleus to the ipsilateral and contralateral Edinger-Westphal nuclei. Decussating fibers traverse from the ipsilateral pretectal nucleus to the contralateral Edinger-Westphal nucleus within the posterior commissure. The Edinger-Westphal nucleus is the site of parasympathetic preganglionic neurons. These neurons project fibers within the oculomotor nerve to synapse on parasympathetic postganglionic (para/post) neurons in the ciliary ganglion. From the ciliary ganglion, para/post fibers project to innervate the sphincter of the iris. When light enters one eye, both pupils constrict: a phenomenon called the consensual response. As with many patients suffering from MS, this patient’s injury is isolated to only one optic nerve. Thus, light entering the affected eye (in this case, the left one) will not be transmitted to the midbrain, and neither pupil will respond. Light in the contralateral eye, however, will be transmitted normally to the midbrain, and the response will be transmitted along the unaffected oculomotor nerves to constrict the pupils of both eyes.

Educational Objective: The afferent limb of the light reflex pathway is the optic nerve; the efferent limb lies in parasympathetic fibers of the oculomotor nerve. When an optic nerve is damaged, light in that eye will cause neither pupil to constrict (the nerve can’t sense the light); however, light in the contralateral eye will cause both pupils to constrict (because the motor function of the iris is conserved).

57

Page 58: Anat 50q.doc

USMLE WORLD STEP 1 ANATOMY

A. Fibular (lateral) collateral B. Tibial (medial) collateral C. Patellar D. Anterior cruciate E. Posterior cruciate F. Oblique popliteal

Explanation:The anterior cruciate ligament (ACL) connects the lateral femoral condyle to the anteromedial tibial head. The ACL prevents anterior motion of the tibia with respect to the femur. Damage to the ACL can be assessed on physical examination with the anterior drawer test, a maneuver that involves flexing the lower extremity to approximately 900 and pulling the tibia anteriorly with the femur held fixed. Laxity indicates ACL injury. This patient appears to have sustained force on the anterior tibia greater than the ACL could bear, causing ligament rupture. Athletes who play contact sports are at risk for the “unhappy triad” of knee injuries, which includes simultaneous damage to the tibial (medial) collateral ligament, medial meniscus, and ACL. These injuries generally result from a forceful blow to the lateral knee while the lower extremity is fully extended and the foot is rigidly planted. Extreme force to the lateral knee causes tearing of the tibial (medial) collateral ligament. Because this ligament is attached to the medial meniscus, there is frequently also tearing of the medial meniscus. The ACL can also be injured by this mechanism due to anterior and medial movement of the tibia with respect to the femur.

58

Q NO 44: A 12-year-old soccer player presents to the emergency room with acute right knee pain after sustaining a kick injury to an extended leg. The trauma caused anterior displacement of her tibia with respect to her femur. Which of the following ligaments was most likely injured?

Page 59: Anat 50q.doc

USMLE WORLD STEP 1 ANATOMY

(Choice A) The fibular (lateral) collateral ligament is extremely strong and is rarely injured. Injury o this ligament would require a forceful blow to the medial knee while the leg is extended. (Choice C)The patellar ligament is the inferior projection of the quadriceps femoris tendon. Injury causes difficulty with leg extension. (Choice E)The posterior cruciate ligament extends from the medial condyle of the femur to the posterior head of the tibia. It functions to prevent posterior movement of the tibia with respect to the femur. Damage is assessed using the posterior drawer test. (Choice F)The oblique popliteal ligament is a wide fibrous band that connects the posterior distal femur to the posterior proximal tibia. It has small openings for nerves, vessels, and muscle insertions.

Educational Objective: The anterior cruciate ligament (ACL) can be damaged by forceful anterior motion of the tibia with respect to the femur. ACL injury can occur as part of the “unhappy triad,” which also includes injury to the medial meniscus and medial collateral ligament.

59

Page 60: Anat 50q.doc

USMLE WORLD STEP 1 ANATOMY

A. Thyroidectomy B. Anterior dislocation of the shoulder joint C. Violent stretch between the head and the shoulder D. Clavicular fracture E. Mastectomy

Explanation:The patient described in the question stem is exhibiting symptoms consistent with paralysis of the serratus anterior muscle due to long thoracic nerve injury. Paralysis of the serratus anterior causes two classic signs. First, when a patient is asked to press anteriorly against a wall scapular “winging” can be observed. A winged scapula results from an inability of the serratus anterior to hold the medial border and inferior angle of the scapula against the posterior chest wall. Second, patients with serratus anterior paralysis are unable to abduct the arm higher than the horizontal position. The deltoid and supraspinatus muscles abduct the arm up to the horizontal position but at that point the action of the serratus anterior is required to rotate the glenoid cavity superiorly thereby allowing complete abduction of the arm over the head. The long thoracic nerve injury can occur during penetrating trauma or iatrogenically during axillary lymph node dissections as may occur during a radical mastectomy. (Choice A) Thyroidectomy carries many risks due to the complicated anatomy of the midline neck. Nerve injuries that can occur with this procedure include damage to the recurrent laryngeal nerve during ligature of the inferior thyroid artery and damage to the external branch of the superior laryngeal nerve during ligature of the superior thyroid artery. (Choice B) Anterior dislocation of the shoulder joint or a fracture of the neck of the humerus can injure the axillary nerve, resulting in paralysis of the deltoid and teres minor and some sensory loss to the upper lateral arm. (Choice C) Violent stretch between the head and the shoulder can occur during delivery or by trauma in the adult. This results in damage to the upper trunk of the brachial plexus and the classic Erb-Duchenne palsy. This palsy results from damage to the musculocutaneous and suprascapular nerves and causes a waiter’s tip posturing of the arm (shoulder adducted, arm pronated, and elbow extended). (Choice D) Clavicular fracture does not cause any characteristic nerve palsies, but it is important to note that these fractures typically occur in the middle third of the clavicle due to the strength of the ligamentous structures at either end of this bone.

Educational Objective: Mastectomy is a commonly tested cause of long thoracic nerve injury, but any trauma or surgery in the axillary region is at risk of damaging this nerve. Injury to this nerve causes winging of the scapula and inability to abduct the shoulder past 90 degrees.

60

Q NO 45: A 34-year-old Caucasian female has difficulty abducting her right arm past the horizontal position. An abnormal prominence of the right inferior scapular angle is also observed. Which of the following is the most likely cause of this patient’s condition?

Page 61: Anat 50q.doc

USMLE WORLD STEP 1 ANATOMY

A. Right ventricle B. Right atrium C. Left ventricle D. Left atrium E. Pulmonary artery

Explanation:Given her history of dyspnea and cardiomegaly, this patient’s dysphagia maybe second arqto a cardiac cause. Cardiovascular dysphagia is uncommon, but when it does occur, it is usually the result of left atrial enlargement. (The left atrium is located posteriorly, directly overlying the esophagus.) Left atrial enlargement most commonly occurs in the setting of mitral stenosis and left ventricular failure. Increasing pressures in the left atrium cause distention of this chamber and pressure on the underlying esophagus. (Choice A) The right ventricle is located on the right lateral aspect of the heart and abuts the right lung. (Choice B)The right atrium is located on the anterior and superior right aspect of the heart and receives blood from the inferior and superior venae cavae as well as the coronary sinus. (Choice C)The left ventricle is located on the left lateral side of the heart and abuts the left lung. (Choice E)The pulmonary artery emerges from the right ventricle and branches into the right and left pulmonary arteries below the arch of the aorta.

Educational Objective: Cardiovascular dysphagia can result from pressure on the esophagus by a dilated left atrium. The left atrium is commonly enlarged in patients with mitral stenosis and left ventricular failure.

61

Q NO 46: A 65-year-old female is evaluated for progressive dyspnea and cardiomegaly. She also complains of occasional dysphagia. Her dysphagia is potentially explained by enlargement of the:

Page 62: Anat 50q.doc

USMLE WORLD STEP 1 ANATOMY

A. Genitofemoral B. Iliohypogastric C. Lateral femoral cutaneous D. Obturator E. Pudendal F. Inferior gluteal

Explanation:A pudendal nerve block is one method of providing anesthesia during childbirth, and this is the block that is described in this scenario. Sometimes this method is used if a woman has progressed too far in labor to receive epidural anesthesia. The pudendal nerve is derived from the 32 - 34 nerve roots and provides sensory innervation to the perineum and genitals (of both sexes) as well as motor innervation to the sphincter urethrae and the external anal sphincter. When administering a pudental nerve block the physician generally palpates intravaginally for the ischial tuberosity and attempts to administer the anesthetic agent in that location. (Choice A) The genitofemoral nerve originates at L1 and L2 and courses on the anterior surface of the psoas muscle. It splits into the genital and femoral branches, innervating the scrotum / labia majora and cutaneously innervating the femoral triangle, respectively. (Choice B) The iliohypogastric nerve is derived from T12 / L1 and courses in that dermatome to innervate the skin overlying the iliac crests. (Choice C) The lateral femoral cutaneous nerve is derived from L2 and L3. It courses deep to the inguinal ligament to innervate the skin on the anterolateral thigh. (Choice D) The obturator nerve is derived from LB and L4 and serves to provide motor innervation to the medial thigh (adductors). (Choice F) The inferior gluteal nerve is derived from L5 - 32 roots and provides motor innervation to the gluteus maximus.

Educational Objective: A pudendal nerve block can be performed by injecting anesthetic intravaginally in the region of the ischial spine. Blocking the pudental nerve provides anesthesia to the majority of the perineum; additional blockade of the genitofemoral and ilioinguinal nerves would provide complete perineal and genital anesthesia.

62

Q NO 47: A 23-year-old female is examined by an obstetrician in the delivery room. Starling intravaginally, he injects lidocaine solution near the tip of the ischial spine. Which of the following nerves is most likely blocked by the injection?

Page 63: Anat 50q.doc

USMLE WORLD STEP 1 ANATOMY

A. Ethmoid cells B. Maxillary sinus C. Frontal sinus D. Sphenoid sinus E. Inferior conchae

Explanation:The orbit is bound superiorly by the thick orbital plate of the frontal bone and laterally by the thick bone of the zygoma and greater and lesser sphenoid wings. In contrast, the orbital floor is composed of a very thin layer of bone that separates the orbit from the air-filled maxillary sinus. Similarly, the orbit’s medial wall is composed of the thin ethmoid and lacrimal bones, which separate it from the ethmoid air cells. Blunt trauma to the eye causes a rapid increase in pressure that typically does not rupture the globe but is transmitted posteriorly into the orbit. The weakest plates of bone in the orbit, the medial and inferior walls, are common sites of fracture. Fracture is typically evident on radiographic imaging, and fluid (blood) can usually be visualized in the normally air-filled adjacent sinuses. On the right side of the image above (corresponding to the patient’s left side), the maxillary sinus (below the orbital floor) is filled with fluid. (Choice A) The ethmoid air cells are medial to the orbit. Here they are unaffected, but they can be affected in blowout fractures of the orbit’s medial wall. (Choice C) The frontal sinus is not pictured above. It lies in the medial part of the supraorbital frontal bone. (Choice D) The sphenoid sinuses (not pictured above) lie anterior to the optic chiasm and are not in close relation to the orbit. (Choice E) The inferior nasal conchae form part of the nasal turbinate system. They are not air-filled sinuses.

Educational Objective: Blunt trauma to the globe can cause orbital blow out fractures. These fractures most commonly involve the medial or inferior orbital walls, because the bone bordering the ethmoid and maxillary sinuses is thin.

63

Q NO 48: A 30-year-old male presents to the emergency room after being involved in a street fight. There is significant soft tissue swelling around the left eye. The imaging study above reveals a left orbital floor fracture with fluid accumulation in the:

Page 64: Anat 50q.doc

USMLE WORLD STEP 1 ANATOMY

A. Cystic duct B. Common bile duct C. Duodenum D. Jejunum E. Ileum

Explanation:This patient is most likely suffering from gallstone ileus, a condition that occurs in patients with longstanding cholelithiasis (often middle-aged to elderly females). A large (typically 2.5cm or greater) gallstone causes the formation of a cholecystenteric fistula between the gallbladder and adjoining gut tissue due to persistent pressure on these tissues. The fistula ultimately allows passage of the gallstone into the small bowel, and the resulting communication between the gallbladder and small bowel allows intestinal gas to enter the gallbladder and biliary tree. Often this gas can be observed on abdominal X-rays. After entering the small bowel, the gallstone typically passes freely through the duodenum jejunum and ileum until it encounters the ileocecal valve. The ileocecal valve typically handles material in a liquid state, and the large gallstones associated with gallstone ileus are generally unable to pass through this valve. Obstruction of the small bowel at the ileocecal valve leads to the classic signs and symptoms of small bowel obstruction including abdominal pain and distention, vomiting, tenderness to palpation and tinkling bowel sounds. The diagnosis can be made with an upper GI series to demonstrate the cholecystenteric fistula and small bowel obstruction. Treatment is by surgical removal of the offending stone. Cholecystectomy and repair of the fistula are rarely necessary. (Choices A and B) Obstruction of the cystic duct or common bile duct by a gallstone is referred to as choledocholithiasis. This can lead to biliary colic, jaundice and cholangitis. Signs and symptoms of cholangitis include fever, right upper quadrant abdominal pain, and an inspiratory pause during right upper quadrant palpation (Murphy’s sign). Free air is typically not seen in the biliary tree. Right upper quadrant ultrasound will reveal stones in the biliary ducts. (Choices C and D)The caliber of the duodenum and jejunum is typically sufficient to allow the passage of the larger gallstones that cause gallstone ileus.

Educational Objective: Gallstone ileus results from the passage of a large gallstone (typically greater than 2.5 cm) through a cholecystenteric fistula into the small bowel where it ultimately causes obstruction at the ileocecal valve. Gas is seen within the gallbladder and biliary tree on abdominal X-ray due to the presence of the fistula, and patients present with signs and symptoms of small bowel obstruction.

64

Q NO 49: A 56-year-old female with a long history of gallstones presents to the emergency room complaining of cramping mid-abdominal pain, abdominal distention and vomiting for the last 12 hours. Abdominal X-ray reveals air in the gallbladder and biliary tree. In which of the following sites is a gallstone most likely lodged?

Page 65: Anat 50q.doc

USMLE WORLD STEP 1 ANATOMY

A. Suprapatellar bursa B. Prepatellar bursa C. Anserine bursa D. Semimembranous (popliteal) bursa E. Gastrocnemius bursa

Explanation:The prepatellar bursa is located between the patella and the overlying skin and prepatellar tendon. It is lined by synovium and contains very little fluid. Its function is to diminish friction and ensure maximal range of motion at the knee. Chronic trauma from repeated kneeling is one cause of prepatellar bursitis, also called “housemaid’s knee.” It is common in roofers, plumbers and carpet layers. Symptoms of prepatellar bursitis include knee pain, swelling, redness and inability to flex the knee on the affected side. The symptoms are usually relieved by rest. Physical examination reveals tenderness to palpation, erythema, crepitance, and fluctuant edema over the lower pole of the patella. (Choices A, D and E) Prolonged kneeling is not associated with inflammation of the suprapatellar, popliteal or gastrocnemius bursae. Popliteal and gastrocnemius bursitis can be associated with the formation of Baker’s cysts. (Choice C) Anserine bursitis (pes anserinus bursitis) presents with pain along the medial aspect of the knee. It results from overuse in athletes or from chronic trauma in patients with heavy body habitus. There is typically tenderness to palpation approximately 4 cm distal to the anteromedial joint margin of the knee.

Educational Objective: Repeated and prolonged kneeling can cause prepatellar bursitis. Dubbed “housemaid’s knee,” today it is most commonly seen in roofers, carpenters and plumbers. Signs and symptoms of prepatellar bursitis include knee pain, erythema, swelling and inability to knee on the affected side.

65

Q NO 50: A 46-year-old gardener presents to your office with knee pain. He reports that he spends hours on his knees several times per week while gardening. Which of the following bursae do you most expect to be affected in this patient?